Home
This site is intended for healthcare professionals
Advertisement
Share
Advertisement
Advertisement
 
 
 

Summary

Join us for the ICSM MedEd Year 3 Mock 2025 session, a valuable training resource for the Year 3 Imperial Written Exam! Crafted by expert lecturers, this comprehensive session includes 150 mock questions mirroring the actual exam content. Further enrich your understanding by attending a live discussion held on MedAll, where difficult questions are discussed in detail by faculty members. This platform opens up opportunities for Q&A to clarify all your exam-related queries. Don't miss this unique learning opportunity to equip yourself with the requisite knowledge and prepare for the challenging examination. A helpful guide for aspiring medical professionals who aim to excel in their academics!

Generated by MedBot

Description

This is the official 2025 Year 3 MedEd Mock!

We will be hosting a Q&A session based on the mock on the 2nd of June 2025, please do sign up on MedAll to attend. Please submit and questions you have prior to the session via this form: https://forms.cloud.microsoft/e/9b9WixGkjV

Learning objectives

  1. By the end of the session, learners will be able to differentiate between various cardiac and respiratory conditions using patient symptoms and test results, to appropriately diagnose and manage patients.
  2. Participants should be able to recognize the signs and symptoms of peripheral vascular disorders and accurately diagnose and treat such conditions following the teaching session.
  3. Learners will be able to effectively assess and manage acute emergencies such as suspected deep vein thrombosis, and acute limb ischaemia following the session.
  4. By the conclusion of the teaching session, learners should be able to diagnose and initiate appropriate management plans for gastrointestinal diseases such as esophageal cancer, reflux disease, or peptic strictures.
  5. The teaching session aims to enhance the learners' understanding and competency in diagnosing and managing various urological, ophthalmological, and neurological conditions, based on patient symptoms, examination findings, and the results of diagnostic tests.
Generated by MedBot

Similar communities

View all

Similar events and on demand videos

Computer generated transcript

Warning!
The following transcript was generated automatically from the content and has not been checked or corrected manually.

ICSM MedEd Year 3 Mock 2025 Written by: All Year 3 Written Series Lecturers from the 2025 Series Edited by: Shruti Rajendra, Sachi Parikh, Ayesha Duhra, Michael Song, Lizzie Varaksina and Vaish Ragavan This mock contains 150 questions, similar to the Year 3 Imperial Written Exam. Answers and explanations are at the end of this document. A live session held on MedAll on Monday 2nd June 2025 @6:30pm and will run through the most difficult questions and address any other exam-related questions. Please submit any questions you have for this session via the following QR: Any other questions, please contact us at: meded@ic.ac.uk, sachi.parikh21@imperial.ac.ukQuestion 1 A 65-year-old man presents to the GP with worsening shortness of breath and difficulty sleeping flat at night. He mentions needing three pillows to sleep and waking up gasping for air. He also notes fatigue and occasional palpitations. He has a history of hypertension and type 2 diabetes. On examination, there are fine inspiratory crackles at the lung bases, an S3 gallop, and mild ankle oedema. Blood tests reveal an elevated BNP. Echocardiography shows an ejection fraction of 35%. What is the most appropriate initial step in management? A. Increase bisoprolol dose B. Start furosemide C. Start digoxin D. Add isosorbide mononitrate E. Refer for urgent valve surgery Question 2 A 70-year-old man presents with a 6-month history of worsening left calf pain on walking. He can now only manage 50 metres before needing to stop. The pain is relieved by rest. On examination, the left foot is cool with diminished dorsalis pedis and posterior tibial pulses. There is thinning of the skin and reduced hair growth on the shin. ABPI is 0.55 on the left and 1.0 on the right. What is the most likely diagnosis? A. Acute limb ischaemia B. Critical limb ischaemia C. Chronic limb ischaemia with intermittent claudication D. Venous insufficiency E. Diabetic neuropathy Question 3 A 56-year-old man presents to A&E with a red, swollen, and painful left lower leg that has rapidly worsened over the past 12 hours. He is febrile (38.9°C), tachycardic, and hypotensive. On examination, the leg is erythematous, tense, and exquisitely tender out of proportion to appearance. There is a small patch of skin discoloration and some blistering. What is the most appropriate next step in management? A. Start oral flucloxacillin and review in 48 hours B. Refer urgently for surgical debridementC. Start topical corticosteroids D. Administer intramuscular benzathine penicillin E. Order a Doppler ultrasound to assess for DVT Question 4 A 60-year-old male presents to his GP with dysphagia initially associated with solids and now liquids over the past few months. He has also experienced unintentional weight loss of 5 kg and occasional reflux symptoms over this time. His past medical history includes gastroesophageal reflux disease (GORD) for several years, but he does not take any regular treatment. What is the most likely diagnosis? A. Achalasia B. Barrett’s oesophagus C. Oesophageal adenocarcinoma D. Oesophageal squamous cell carcinoma E. Peptic stricture Question 5 A 63 M presents with frank haematuria, but report no changes to passing urine. They report feeling quite tired the last few months and their clothes have felt looser. They are an ex- smoker with a 46 pack-year history. Examination: HR 72 bpm, BP 165/80 mmHg, O2 sats 96% on air and temp 37.2ºC. On palpation he has tenderness in the left renal angle and a ballotable mass on the left side of the abdomen. On digital rectal exam his prostate is not enlarged and non-tender. What is the most likely diagnosis? A. Urolithiasis B. STI C. Renal cell carcinoma D. Lower UTI E. Pyelonephritis Question 6 A 71-year-old man presents to the ophthalmology clinic with a 6-month history of progressive difficulty navigating stairs and bumping into objects, especially on either side of his field of vision. He denies eye pain, diplopia, or previous visual problems. Visual acuity isnormal. Visual field testing reveals a bitemporal hemianopia. Fundoscopy is unremarkable, and there are no focal neurological deficits on examination. Which of the following is the most likely underlying cause of his symptoms? A. Retinal detachment B. Multiple sclerosis C. Pituitary adenoma D. Optic neuritis E. Normal pressure hydrocephalus Question 7 A 24-year-old man collapses suddenly while playing football. Bystanders report he lost consciousness without warning. Despite prompt CPR, he is pronounced dead at the scene. A post-mortem reveals marked asymmetric septal hypertrophy of the left ventricle. His parents mention a paternal uncle who died suddenly in his 30s. Which investigation would have been most useful in identifying this condition during life? A. ECG B. Chest X-ray C. CT coronary angiogram D. Transoesophageal echocardiography (TOE) E. Transthoracic echocardiography (TTE) Question 8 A 43-year-old woman is admitted to A&E with severe right upper quadrant pain. It came on last night and has not subsided. She has experienced similar pain in the past after eating fast food, however, this pain is usually less intense. On examination she is jaundiced. LFTs reveal: Bilirubin 59 micromol/L (3-17) AST: 47 IU/L (5-35) ALT: 49 IU/L (5-35) ALP: 430 IU/L (30-150) What is the most likely diagnosis? A. Gallstones B. Alcoholic hepatitis C. Viral hepatitis D. Hepatocellular carcinoma E. Gilbert's syndromeQuestion 9 A 35-year-old African-Caribbean woman presents with a 3-week history of dry cough, shortness of breath, and fatigue. She also mentions red, tender nodules over her shins and mild joint pain. On examination, she is afebrile with no respiratory distress. Chest X-ray shows bilateral hilar lymphadenopathy Which of the following is the most likely diagnosis? A. Tuberculosis B. Sarcoidosis C. Hodgkin lymphoma D. Systemic lupus erythematosus E. Rheumatoid arthritis Question 10 A 19-year-old university student presents to the GP with a 7-day history of fever, sore throat, and fatigue. On examination, he has enlarged tonsils with exudate, cervical lymphadenopathy, and mild splenomegaly. A monospot test is positive. He is not immunocompromised and has no difficulty swallowing or signs of airway obstruction. What is the most appropriate management? A. Oral amoxicillin B. Oral corticosteroids C. Supportive care with fluids and rest D. Antiviral therapy with acyclovir E. Immediate hospital admission and IV antibiotics Question 11 A 36-year-old woman presents with fatigue, weight loss, dizziness, and increased skin pigmentation. She also mentions frequent episodes of nausea and abdominal pain. Her blood pressure is 90/60 mmHg. Blood tests reveal hyponatraemia and hyperkalaemia. An early morning cortisol is low, and ACTH is elevated. What is the most likely diagnosis? A. Cushing’s disease B. Primary adrenal insufficiency (Addison’s disease) C. Secondary adrenal insufficiency D. Phaeochromocytoma E. SIADHQuestion 12 You are the FY2 on the post-natal ward. You are called to see a 28 year old woman, complaining of a headache. She describes it as stating from the back of her head and 7/10 on the pain scale. She feels nauseous. She has no vision changes or focal neurology. She had given birth vaginally the day before and was given epidural to manage her pain. She has no relevant past medical history. This is her first time giving birth. What is the most likely cause of her headache? A. Migraine B. Medication overuse headache C. Intracerebral haemorrhage D. Post-dural tap E. Tension type headache Question 13 A 26-year-old woman presents to her GP with a 3-day history of pain when passing urine and urinary frequency. She has not noticed any haematuria or foul-smelling urine. She has no flank pain, fever, rigors, nausea or vomiting. She has no relevant medical history. She is allergic to penicillin. Her urine dipstick is positive for nitrates and leukocytes and a sample has been sent for a urine MC+S. A pregnancy test is negative. What is most appropriate management? A. Amoxicillin B. Nitrofurantoin, 3 day course C. Nitrofurantoin, 7 day course D. Advise her to drink cranberry juice E. Gentamicin Question 14 A 79-year-old woman presents with intermittent abdominal discomfort, constipation, and low mood. Her past medical history includes hypertension and diabetes mellitus. Blood tests reveal the following results: • Calcium: 3.3 mmol/L (normal range: 2.1–2.6 mmol/L) • Phosphate: 0.9 mmol/L (normal range: 0.8–1.4 mmol/L) • Alkaline Phosphatase (ALP): 185 IU/L (normal range: 25–110 IU/L) • Parathyroid Hormone (PTH): 18.2 pmol/L (normal range: 1.5–7.5 pmol/L) What is the most likely cause of this patient’s presentation? A) MalignancyB) Pseudohypoparathyroidism C) Primary hyperparathyroidism D) Secondary hyperparathyroidism E) Tertiary hyperparathyroidism Question 15 An 18-year-old woman presents to her GP with a one week history of a sore throat and muscle aches. She has recently started university. The GP has identified warm, tender nodules bilaterally behind the sternocleidomastoid muscle and her tonsils are enlarged on examination. What is the most appropriate investigation to determine the cause of her symptoms? A. Throat swab for culture and sensitives B. Mantoux test C. Monospot D. EBV serology E. LFTs Question 16 A 32-year-old woman presents with confusion, fever, and fatigue. On examination, she has petechiae and mild jaundice. Blood tests reveal anaemia, thrombocytopenia, raised urea, and schistocytes on blood smear. Coagulation studies are normal. What is the next best step in management? A. Intravenous immunoglobulin (IVIg) B. Fresh frozen plasma C. Desmopressin D. Plasma exchange E. Platelet transfusion Question 17 A 55-year-old male presents to their GP after noticing a change on the left side of his face but can still raise his eyebrows. He mentions that it happened sudden but reports there has been no pain or itchiness. The GP notes that the left pupil is dilated. The patient has hypertension and a 15-year pack history. What is the most likely diagnosis? A. Bell’s palsy B. Temporal arteritisC. Multiple sclerosis D. Ischaemic stroke E. Ramsay Hunt syndrome Question 18 A 60-year-old woman presents with a swollen, tender left leg. She recently underwent knee replacement surgery. On examination, the leg is warm and erythematous, with pitting oedema up to the mid-thigh. Wells score is 3. D-dimer is elevated. What is the most appropriate next investigation? A. CT venogram B. MR venogram C. Venous duplex ultrasound D. Repeat D-dimer in 24 hours E. Contrast-enhanced CT angiogram Question 19 A middle-aged male comes in to the GP with a background of facial pain and nasal congestion. The patient says the pain is mostly at the front of his face and his symptoms started 3 months ago. The GP elicits that the nasal discharge has always been clear and the patient has lost his sense of smell. Given the most likely diagnosis, what is the treatment option that should be considered? A. Surgery B. Nasal irrigation C. Antibiotics D. Intranasal steroids E. Anterior rhinoscopy Question 20 A 68-year-old man presents with fatigue and early satiety. On examination, there is a palpable mass in the left upper quadrant. Blood tests reveal pancytopenia. A bone marrow aspirate results in a "dry tap." What is the most likely diagnosis? A. Polycythaemia vera B. Myelodysplastic syndrome C. MyelofibrosisD. Acute myeloid leukaemia E. Chronic myeloid leukaemia Question 21 A 60-year-old man presents with a 6-month history of resting tremor in his right hand, bradykinesia, and cogwheel rigidity. His symptoms improve with movement. What is the most likely diagnosis? A. Essential tremor B. Parkinson’s disease C. Multiple system atrophy D. Progressive supranuclear palsy E. Huntington's disease Question 22 A 31-year-old man presents to the GP with mild shortness of breath, a low-grade fever and a non-productive cough. He does not smoke. He has noticed some weight loss but thought this was due to change in diet from when he was in India. Which of the following is crucial for making the diagnosis? A. CT chest B. Sputum microscopy with gram stain C. HIV test D. Urine analysis E. Chest x-ray Question 23 A 37-year-old man presents to ED with his girlfriend. He seems drowsy and is complaining of a headache and muscle pains. His girlfriend reports that he has been ‘acting funny’ and ‘not himself’ for the past few days. She says that he easily gets upset and is often dazed and forgetful. He had no relevant past medical history. No known drug allergies. No recreational drug use. He works as an accountant. He lives with his girlfriend who is non-symptomatic. A recent set of observations is listed below: • RR 22 • HR 108 • BP 115/87 • SpO2 97% on room air • Temperature 38.9 CWhat is the most likely organism causing this man’s presentation? A. Cryptococcus neoformans B. Neisseria meningitidis C. Mycobacterium tuberculosis D. Toxoplasma gondi E. Herpes simplex virus Question 24 A 50-year-old woman presents to the emergency department with muscle cramps and tingling in her fingers. She experiences facial muscle spasms when the pre-auricular region is tapped. Her medical history includes a total thyroidectomy performed for papillary thyroid cancer 6 months ago. Laboratory tests reveal: • Serum Calcium: 1.6 mmol/L (normal range: 2.2–2.6 mmol/L) • Parathyroid Hormone (PTH): 0.5 pmol/L (normal range: 1.5–6.5 pmol/L) What is the most appropriate initial management for this patient? A. High-dose vitamin D supplementation B. Oral calcium carbonate C. Administration of loop diuretics D. Intravenous calcium gluconate E. Administration of thiazide diuretics Question 25 A 69 M arrives at their GP due to painless blood in their urine. They recently began complaining of a mild testicular ache. He is a heavy smoker smoking 60 cigarettes a day for 47 years. On examination he is cachectic and has a palpable mass on the left side of the abdomen. His bloods reveals anaemia. What sign in a testicular examination would be indicative of the most likely diagnosis? A. Normal B. “Bag of worms” C. Loss of the cremasteric reflex D. Swelling and tenderness eased by elevating testes E. Soft, fluctuant lumpQuestion 26 A 65-year-old male presents with a three-month history of epigastric pain, early satiety, and unintentional weight loss of 8 kg. He also reports nausea and occasional melaena. His past medical history includes chronic H. pylori infection, which was treated five years ago. On examination, he appears pale with a palpable lump in his left supraclavicular fossa. Routine blood tests reveal iron-deficiency anaemia. What is the most appropriate investigation to confirm the likely diagnosis? A. Barium swallow B. Chest X-ray C. Colonoscopy with biopsy D. CT abdomen and pelvis E. Oesophagogastroduodenoscopy (OGD) with biopsy Question 27 A 72-year-old man is brought to A&E after being found drowsy and confused. He has COPD and chronic kidney disease. His daughter reports he has been vomiting and lethargic for the past 2 days. He is hypoventilating. ABG: pH: 7.17 (7.35 – 7.45) PaCO₂: 8.5 kPa (4.7 – 6) HCO₃⁻: 15 mmol/L (22 – 26) Na⁺: 140 mmol/L (135 – 145) Cl⁻: 101 mmol/L (95 – 105) Urea: 19 mmol/L (2 – 7) Creatinine: 265 μmol/L (55 – 120) Lactate: 1.3 mmol/L (0.5 – 2.2) Glucose: 5.2 mmol/L (3.9 – 5.6) What is the most likely acid-base disturbance? A. Metabolic acidosis with respiratory compensation B. Respiratory acidosis with renal compensation C. Mixed respiratory and metabolic acidosis D. Metabolic alkalosis with respiratory acidosis E. Normal anion gap metabolic acidosis Question 28 A 25-year-old tall, thin man presents with sudden-onset pleuritic chest pain and shortness of breath. On examination, he is stable, but breath sounds are absent on the left side. Chest X-ray confirms a 3.5 cm rim of air between the lung margin and chest wall. What is the most appropriate next step in management? A. High-flow oxygen and observe for 4 hours B. Urgent needle decompression C. Immediate chest drain insertion D. Needle aspiration E. IV antibiotics and admit under respiratory Question 29 A 10-month-old infant is brought to the GP with a 3-day history of high fever (up to 40°C). The child is otherwise active and feeding well. On the fourth day, the fever resolves abruptly, and a pink, non-itchy maculopapular rash appears on the trunk and spreads to the neck and extremities. The child is alert and playful. What is the most likely diagnosis? A. Measles B. Scarlet fever C. Rubella D. Roseola infantum E. Erythema infectiosum Question 30 A 68-year-old woman with a history of varicose veins presents to the GP with a chronic ulcer just above her medial malleolus. It has an irregular border, is shallow with granulation tissue, and is surrounded by hyperpigmented, indurated skin. Peripheral pulses are present and strong. What is the most likely diagnosis? A. Arterial ulcer B. Diabetic foot ulcer C. Venous ulcer D. Pressure ulcer E. Pyoderma gangrenosum Question 31 A 54-year-old man presents to A&E after waking up with left-sided facial weakness. On examination, he has weakness of the lower left face but can still raise both eyebrowssymmetrically. There are no signs of limb weakness or sensory changes. He has a history of poorly controlled hypertension. A CT brain is ordered. Which of the following best explains his presentation? A. Lower motor neuron facial nerve lesion B. Middle cerebral artery infarct C. Brainstem glioma D. Parotid gland tumour E. Ramsay Hunt Syndrome Question 32 A 68-year-old presents to his GP with difficulty initiating micturition and dribbling. A DRE is performed which reveals a symmetrically enlarged prostate. He has presented to his GP with the same symptoms and has attempted to make changes to his lifestyle, such as reducing his fluid intake at night, but feels that his symptoms have not improved. The patient is referred for a blood test in two weeks, the results are listed below: • PSA 2.3 (<4.0 ng/mL) • Na 138 (135 - 145 mmol/L) • K 4.1 (3.5 - 5.1 mmol/L) • Cl 97 (95 - 105 mmol/L) • Urea 3.5 (1.7 - 8.3 mmol/L) • Creatinine 75 (66 - 112 umol/L) • Serum adjusted Ca 2.27 (2.20 - 2.60 mmol/L) • ALP 35 (30-130 IU/L) What is the most appropriate management? A. Refer to urology on 2 week wait cancer pathway B. Start patient on tamsulosin and review symptoms in 4 weeks C. Refer the patient for a TURP D. Start oxybutynin E. Refer the patient for long-term catheterisation Question 33 A 48-year-old woman with a history of chronic alcohol use presents with fatigue, exertional breathlessness, and mild ankle swelling. On examination, she has a displaced apex beat, a soft S3, and bibasal crackles. CXR shows an enlarged, globular heart. Echocardiography reveals a dilated left ventricle with globally reduced systolic function. What is the most likely diagnosis? A. Hypertrophic cardiomyopathy B. Restrictive cardiomyopathyC. Constrictive pericarditis D. Dilated cardiomyopathy E. Pulmonary embolism Question 34 A 35-year-old man presents to his GP with a midline neck lump. The GP identifies it as a thyroglossal cyst. What are the expected examination findings? A. Lump moves up on tongue protrusion B. Lump is hard and feels craggily C. Lump is rubbery, warm and tender on palpation D. Lump is more apparent when patient blows out air E. Lump is diffuse and grossly symmetrical with a palpable thrill Question 35 A 63-year-old female arrives at A&E with a sudden onset of left sided facial weakness and sensory loss. The patient’s speech is muddled but seems to understand what the doctors are saying. Given the most likely diagnosis, what is the next most appropriate step? A. Thrombolysis B. Non-contrast CT C. Aspirin 300mg D. MRI E. CT angiogram Question 36 A 28-year-old woman presents with symmetrical joint pain and morning stiffness lasting >1 hour for 3 months. Anti-CCP is positive. What is the first-line disease-modifying treatment according to NICE? A. Hydroxychloroquine B. Sulfasalazine C. Methotrexate D. NSAIDs E. PrednisoloneQuestion 37 A 53-year-old woman presents to the accident and emergency department with proximal muscle weakness and pain in her limbs. She has noticed a dry mouth in the mornings and is a smoker (40-pack year history). She has a persistent cough and has found her clothes baggier of late. What is the most likely diagnosis? A. Squamous cell lung cancer B. Tuberculosis C. Small cell lung cancer D. Carcinoid tumour E. Adenocarcinoma Question 38 A 68-year-old male presents with abdominal pain, vomiting, distension, and obstipation. He underwent abdominal surgery 10 years ago. Supine AXR shows central dilated bowel loops with valvulae conniventes and multiple air-fluid levels. There is no free air under the diaphragm on erect CXR. His lactate is normal. What is the best initial management step? A. Emergency laparotomy B. Intravenous antibiotics and immediate CT scan C. NBM, NG tube, IV fluids, and close monitoring D. Bowel resection with primary anastomosis E. Sigmoidoscopy for decompression Question 39 A 58-year-old male with a 5-year history of GORD undergoes upper GI endoscopy and is found to have segmental Barrett’s oesophagus measuring 4 cm in length. Biopsies reveal no dysplasia. What is the most appropriate management strategy? A. Immediate endoscopic mucosal resection B. Continue PPI and repeat endoscopy every 6 months C. Endoscopic submucosal dissection D. High-dose PPI and surveillance endoscopy every 2–3 years E. Radiofrequency ablationQuestion 40 A 21-year-old woman presents with prolonged bleeding after a dental procedure. She reports a history of menorrhagia and occasional epistaxis. Labs show normal platelet count, prolonged bleeding time, and prolonged APTT. PT is normal. Which of the following lab findings would support your diagnosis? A. Low factor IX B. Prolonged PT C. Low factor VIII D. Elevated fibrinogen E. Increased platelet count Question 41 A 29-year-old woman presents with fluctuating muscle weakness that worsens throughout the day, particularly affecting her eyes and face. She has bilateral ptosis and diplopia by the evening. Examination shows fatigable weakness, but reflexes and sensation are normal. Which of the following is the most appropriate first-line investigation to confirm the suspected diagnosis? A. CT head B. Serum anti-AChR antibody test C. MRI brain D. Lumbar puncture E. EEG Question 42 A 60-year-old man with longstanding type 2 diabetes complains of numbness in both feet. On examination, he has reduced vibration sense and absent ankle reflexes. What is the most appropriate next step in his management? A. Nerve conduction studies B. Foot X-ray C. Start duloxetine and optimise glycaemic control D. Refer to neurology urgently E. Prescribe vitamin B12 injectionsQuestion 43 A patient presents to A&E with complete hearing loss in their left ear. The patient states that it started 30 minutes ago when they were walking around a park. They mention it was sudden and immediate but otherwise report no symptoms. The patient has no past medical history of note. The doctors conduct their examinations and find the Weber’s test localises to the right ear and the Rinne’s is positive in both. What is the most likely diagnosis? A. Idiopathic B. Acoustic neuroma C. Wax impaction D. Ototoxicity E. Meniere’s disease Question 44 A 10-year-old boy presents to the GP with a 6-month history of wheeze, shortness of breath, and dry cough, which worsens at night and during exercise. He has been using a salbutamol inhaler 2-3 times per week, with only partial relief. His peak flow readings are variable and there is no evidence of acute infection. He is not currently on any preventer medication. What is the most appropriate next step in the management of his asthma? A. Increase salbutamol to regular use B. Add a leukotriene receptor antagonist (LTRA) C. Start a low-dose inhaled corticosteroid (ICS) D. Start a long-acting beta-agonist (LABA) E. Prescribe oral corticosteroids Question 45 An 80-year-old man with dementia is brought to A&E from a care home with confusion, dry mucous membranes, and reduced skin turgor. His carers say he hasn’t been drinking well for the past few days due to agitation. Blood results: Na⁺: 160 mmol/L (135 – 145) Urea: 15.6 mmol/L (2 – 7) Creatinine: 152 µmol/L (55 – 120) Glucose: 6.1 mmol/L (3.9 – 5.6) Serum osmolality: 325 mOsm/kg (275 – 295) Urine osmolality: 850 mOsm/kg (high)What is the most appropriate management? A. Rapid correction with 0.9% saline B. Oral water and high-calorie diet C. Intravenous 5% dextrose with gradual correction D. Intravenous loop diuretics with fluids E. Intravenous hypertonic saline Question 46 A 10-year-old boy is brought to the GP by his mother due to concerns about mul ple pigmented skin lesions. On examina on, he has 8 café-au-lait macules, all greater than 1 cm in diameter, two so , non-tender cutaneous nodules on his torso, and visible freckling in the axillary regions. There is also mild thoracic scoliosis noted, although the child is otherwise well. There is no known family history of similar features, and his development has been age-appropriate. Which of the following is the most appropriate next step in management? A. Arrange MRI of the brain and spine B. Refer to dermatology for skin lesion biopsy C. Refer to clinical gene cs D. Monitor with annual follow-up in primary care E. Ini ate cor costeroid therapy to reduce skin lesions Question 47 An 76-year-old male patient is brought in by ambulance to A&E. The patient has reduced consciousness and his wife mentions that he has been acting differently for the last week. She reports that he has unusual episodes of aggression and has been sleeping more. The patient also was confused for a period but the wife mentions this resolved within a day. The patient has a past medical history of atrial fibrillation and has been noted to be at risk of falls. What is the most likely diagnosis? A. Extradural haemorrhage B. Frontotemporal dementia C. Cranial tumour D. Subdural haemorrhage E. Ischaemic stroke Question 48 A 7-year-old boy is brought to the GP with a 2-day history of sore throat, fever, and difficulty swallowing. On examination, his tonsils are enlarged, erythematous, and covered with whiteexudates. He also has tender cervical lymphadenopathy. There is no cough or nasal congestion. His temperature is 38.7°C. What is the most appropriate next step in management? A. Prescribe amoxicillin B. Prescribe phenoxymethylpenicillin C. Perform monospot test D. Prescribe azithromycin E. Give symptomatic treatment only Question 49 A 6 year old girl presents to the GP accompanied by her father. Her fathers says she has been upset the previous night and this morning. She has been holding the back of her neck and crying. Her father says she is warm to the touch and very reluctant to get out from under her bed covers this morning. He noticed a purple rash on her forearms this morning which didn’t go away when he pressed a glass tumbler against it. She had some coryzal symptoms a week before, which have since settled. Otherwise, she has no past medical history. All her vaccinations are up to date. She has no known drug allergies. What is the most appropriate next step? A. Reassure the father and advise to come back in a week if symptoms persist B. Refer the child to a paediatrician C. Give IM benzylpencillin and call an ambulance D. Give IM benzylpencillin and book a follow-up appointment in two days E. Advise the father to give her over-the-counter paracetamol Question 50 A 65-year-old man presents with pain and stiffness in the shoulders and hips, especially in the mornings. ESR is 70 mm/hr. What is the first-line treatment for polymyalgia rheumatica? A. Naproxen B. Prednisolone 15 mg daily C. Methotrexate D. Hydroxychloroquine E. Physiotherapy Question 51 A 32-year-old woman has unintentional weight loss associated with sweating. She has a diffuse goitre and mild exophthalmos.What is the most appropriate investigation to confirm the diagnosis? A. Anti-thyroglobulin antibody B. Serum thyroglobulin C. Serum thyroid stimulating hormone D. Serum thyroxine E. Thyroid microsomal antibody Question 52 A 55-year-old woman is found to have a platelet count of 900 × 10⁹/L on routine blood tests. She reports occasional headaches and episodes of burning pain in her fingers. There is no evidence of infection or inflammation. What is the most appropriate next investigation? A. Erythropoietin level B. Lymph node biopsy C. Bone marrow biopsy D. JAK2 mutation testing E. ESR and CRP Question 53 A 39-year-old man recently returned from Nigeria presents with fever, chills, and jaundice. On exam, he has splenomegaly and dark-coloured urine. Giemsa-stained thick blood film confirms the diagnosis. What is the most appropriate first-line treatment? A. Oral chloroquine B. IV artesunate C. Artemether-lumefantrine D. IV quinine E. Doxycycline Question 54 A 68-year-old patient presents with an acute, painful red eye. Which of the following signs would not support a diagnosis of acute angle closure glaucoma?A: Vomiting B: Decreased visual acuity C: Constricted pupil D: Haloes E: Hazy cornea Question 55 A 63-year-old man presents to the GP with a history of low mood and headaches. Alongside this, he complains of GI issues and admits to occasionally coughing up blood. He has noticed that he has lost weight in the last month and has a 40-pack year smoking history. What is the most likely diagnosis? A) Carcinoid Syndrome B) Small Cell Lung Cancer C) Squamous Cell Lung Cancer D) Non-Small Cell Lung Cancer E) Pituitary Adenoma Question 56 A 12-year-old girl faints at school and presents to the emergency department. All investigations are normal. What is the mechanism for a vasovagal attack? A: Coronary Artery Spasm B: Thromboembolism C: Peripheral vasodilation and venous pooling D: Histamine release E: Hypoxia Question 57 A 79-year-old man with HFrEF (EF 38%) and recent NSTEMI presents 6 weeks after uptitration of his medications. He reports improved breathlessness and leg swelling, but now feels constantly thirsty and dizzy on standing. Examination: Dry mucous membranes, ↓ skin turgor Clear lungs, no oedema Orthostatic vitals: Lying: BP 120/70, HR 68 Standing: BP 95/52, HR 84Which medication is most likely responsible for his symptoms? A: Atorvastatin B: Ramipril C: Spironolactone D: Furosemide E: Bisoprolol Question 58 A 67-year-old smoker presents with sudden, complete vision loss in his left eye that began one day ago. There is no ocular pain or recent trauma and the patient has a history of hypertension and ankylosing spondylitis. On examination, neither eye appears red. Fundoscopy reveals a small red dot on a pale retina. When light is shone into the right eye, there is a greater constriction of the left pupil than when light is shone directly into the left eye. The response of the right pupil is normal. What is the most likely diagnosis? A: Central retinal artery occlusion B: Optic neuritis C: Anterior uveitis D: Central retinal vein occlusion E: Macular degeneration Question 59 A 58-year-old man is admitted with sepsis secondary to pneumonia. On day 3 of admission, he develops bleeding from venepuncture sites and haematuria. Blood results show thrombocytopenia, prolonged PT and aPTT, low fibrinogen, and elevated D-dimer. What is the underlying pathophysiology? A. Decreased synthesis of clotting factors B. Platelet autoantibodies C. Widespread activation of coagulation cascade D. Factor VIII deficiency E. ADAMTS13 mutation Question 60 A 71-year-old woman with stage 4 CKD is admitted with pneumonia and develops worsening renal function. Over 48 hours, her urine output drops to 100 mL/day. She becomes increasingly confused and dyspnoeic. Blood results: Urea 38 mmol/L, Creatinine 530 µmol/L, K⁺ 6.9 mmol/L, pH 7.20, HCO₃⁻ 13 mmol/L. She hasalready received IV fluids, calcium gluconate, insulin/dextrose, and nebulised salbutamol. What is the most appropriate next step in management? A. Administer loop diuretics to promote urine output B. Insert urinary catheter and monitor fluid balance C. Repeat ECG and continue medical treatment D. Arrange urgent haemodialysis E. Start oral sodium bicarbonate Question 61 A 75-year-old man with stage 4 chronic kidney disease presents with increased SOB and tiredness. His recent blood tests are shown: Hb 110 Man: (135–180) MCV 72 (82–100) Platelets 320×10⁹/L (150–400) WBC 5.6×10⁹/L (4.0–11.0) Na⁺ 138 (135–145) K⁺ 5.0 (3.5–5.0) Bicarbonate 22 (22–29) Urea 8.6 (2.0–7.0) Creatinine 165 (55–120) What is the next most appropriate step in management? A. Send blood for iron studies B. Prescribe oral EPO C. Prescribe IV EPO D. Prescribe IV iron supplementation E. Send blood for EPO level Question 62 A 24-year-old man presents with a sudden onset of raised, itchy, pale wheals on his trunk and limbs. The rash appeared a few hours after eating shellfish and is associated with mild lip swelling but no breathing difficulty. What is the most appropriate initial treatment? A. Oral prednisolone B. Oral antihistaminesC. Epinephrine IM D. Topical corticosteroids E. Intravenous antibiotics Question 63 A 45-year-old presenting with a hard lump in his scrotum is diagnosed with a seminoma. What tumour markers are expected to be raised with his diagnosis? A. b-HCG B. Ca-125 C. Ca-19-9 D. AFP E. PSA Question 64 A 63-year-old man undergoes a CT abdomen for evaluation of abdominal pain, and a 4 cm adrenal mass is incidentally discovered. He is asymptomatic. Blood pressure is 145/90 mmHg. Bloods show normal electrolytes. Further endocrine workup is ordered. Which of the following investigations is essential in the assessment of this adrenal mass? A. Plasma renin and aldosterone B. 8am serum cortisol only C. ACTH stimulation test D. 24-hour urinary catecholamines or plasma metanephrines E. Dexamethasone suppression test only if mass enlarges Question 65 A 69-year-old male presents to the GP with heavy unilateral bleeding from his right nostril. The doctors have attempted the Hippocratic method and cauterisation, but the patient continues to bleed. What is the next most appropriate step? A. Anterior packing and admit to hospital B. Silver nitrate C. Ligation of sphenopalatine artery D. Posterior packing E. Anterior rhinoscopyQuestion 66 A 32-year-old woman presents to her GP with a warm, tender right breast. She is currently breastfeeding her 3 month-old son. The GP notices a 3 cm fluctuant lump on the right breast that is red and tender on palpation. What is the most appropriate management? A. Prescribe co-amoxiclav B. Advise to continue breastfeeding C. Advise to express and discharge breast milk D. Refer to hospital for excision and drainage E. Advise her to use aspirin to manage the pain Question 67 A 62-year-old female presents with dull epigastric pain radiating to the back and unintentional weight loss over the past 2 months. He also reports pale stools and dark urine. On examination, he has scleral icterus with a palpable, non-tender gallbladder. His liver function tests show raised bilirubin, ALP and GGT. Which tumour marker is most commonly associated with the likely diagnosis? A. Alpha-fetoprotein (AFP) B. Carcinoembryonic antigen (CEA) C. CA 19-9 D. CA 125 E. Chromogranin A Question 68 A 65-year-old man presents with slowly progressive pain and stiffness in both knees, especially after activity. On examination, there is crepitus, bony enlargement of the joints, and mild effusion. There is no warmth or erythema. What is the most likely diagnosis? A: Rheumatoid arthritis B: Osteoarthritis C: Psoriatic arthritis D: Reactive arthritis E: GoutQuestion 69 A 60-year-old man reports headaches, pruritus after hot showers, and a ruddy complexion. Blood tests show a haemoglobin of 190 g/L and haematocrit of 0.56. Erythropoietin level is low. What is the most likely diagnosis? A. Chronic myeloid leukaemia B. Dehydration C. Polycythaemia vera D. Renal cell carcinoma E. Essential thrombocythaemia Question 70 Which of the following is the preferred method of definitive diagnosis of mesothelioma? A) CT CAP B) CT Chest C) Bronchoscopy and biopsy D) Chest X-ray E) Thoracoscopy and biopsy Question 71 A 48-year-old man presents to his GP with a 6-month history of progressive shortness of breath, persistent cough, and fatigue. He works as a construction worker throughout his 20- year career. He reports occasional wheezing and tightness in the chest. He also notices that his symptoms worsen during the winter months, and he has been taking over-the-counter cough syrups without much relief. He denies any recent history of infection or fever. His past medical history includes hypertension, and he is a non-smoker. On examination, the patient appears well, but auscultation reveals fine crackles at both lung bases. A chest X-ray shows bilateral small opacities in the mid and lower lung zones. Which of the following is the most likely diagnosis? A. Chronic obstructive pulmonary disease (COPD) B. Asthma C. Silicosis D. Pulmonary tuberculosis E. Idiopathic pulmonary fibrosis (IPF)Question 72 A 24-year-old lady presents to ED with a severe unilateral headache. She had noticed some ‘stars’ in her vision before the headache came on. She has had similar headaches in the last two months, coming every two weeks and lasting for 4 to 8 hours. She normally manages the pain with paracetamol and ibuprofen, however came into ED today as this headache was more painful than usual and she had never had the vision symptoms before. She has a past medical history of asthma. She takes a maintenance therapy of Fostair (beclomethasone with formeterol) 200/6, 2 puffs a day and montelukast 10mg, once every evening. She has no known drug allergies. What is the most appropriate medication to manage her symptoms? A. Sumatriptan B. Propranolol C. Topiramate D. Morphine E. Amitriptyline Question 73 A 72-year-old man is admitted with confusion. Sodium is 123 mmol/L, serum osmolality is low, and urine osmolality and urine sodium are both elevated. He is clinically euvolaemic. Chest X-ray shows a left hilar mass. What is the most likely diagnosis? A. Hypervolaemic hyponatraemia B. Syndrome of inappropriate ADH secretion C. Addison’s disease D. Dehydration E. Primary polydipsia Question 74 A 58-year-old man arrives at the emergency department with severe chest pain. On examination he has dyspnoea and is sweating. He has also vomited 3 times on his way to the hospital. On examination: Blood pressure - 135/90 mmHg. SpO2 - 94% kPa room air. Heart rate - 115 bpm. An ECG is conducted which shows clear ST elevation in leads I, V5-V6. Based on the ECG results. What artery is affected?A: Left anterior descending artery B: Left circumflex artery C: Posterior inter ventricular artery D: Right coronary artery E: Right marginal artery Question 75 A 62-year-old woman who was discharged from the hospital 4 days ago comes to the emergency department due to chest tightness and severe shortness of breath. During the evaluation, the patient becomes unresponsive and goes into cardiac arrest. Despite appropriate lifesaving measures, there is no return of spontaneous circulation and she is pronounced dead. An autopsy is performed which reveals a slit-like tear in the anterior left ventricular wall. What predisposed the patient to the observed cardiac finding? A: Coronary atherosclerosis B: Prolonged alcohol consumption C: Recent viral infection D: Recurrent bacterial pharyngitis E: Repeated blood transfusions Question 76 A 55-year-old woman presents with a persistent dry cough and increasing breathlessness. She is a retired coal miner and has finger clubbing on examination. A chest X-ray is performed:What is the most likely diagnosis? A. Sarcoidosis B. Idiopathic pulmonary fibrosis C. COPD D. TB E. Bronchiectasis Question 77 A 35-year-old man presents with progressive breathlessness and chest discomfort. He reports a recent flu-like illness two weeks ago. On examination, he has a regular pulse, no peripheral oedema, and normal heart sounds. Blood tests show a mildly elevated troponin and CK. ECG shows non-specific ST and T wave changes. You suspect myocarditis. Which symptom is most consistent with this diagnosis? A. Ankle oedema B. Jaundice C. Chest pain worse when lying down D. Ascites E. Kussmaul’s sign Question 78 A 23-year-old woman presents with oral ulcers, joint pain, photosensitive rash, and proteinuria. ANA and anti-dsDNA are positive. What is the most likely diagnosis? A. Rheumatoid arthritis B. Systemic lupus erythematosus C. Sjögren’s syndrome D. Dermatomyositis E. Psoriatic arthritis Question 79 A 29-year-old woman presents with joint pain, photosensitive rash, and fatigue. ANA is positive at high titre, and dsDNA antibodies are also detected. Urinalysis reveals proteinuria and red cell casts. What is the most likely diagnosis?A: Rheumatoid arthritis B: Systemic lupus erythematosus C: Sjogren’s syndrome D: Dermatomyositis E: Antiphospholipid syndrome Question 80 A 30-year-old woman presents to her GP with symptoms of chronic worry, fatigue, muscle tension, and insomnia. She says these symptoms have been present most days for the past 6 months and interfere with her job and social life. She has no depressive symptoms or past psychiatric history. She is hesitant to take medication but wants to try something. What is the most appropriate next step in management? A. Start propranolol B. Offer cognitive behavioural therapy C. Prescribe sertraline D. Recommend lorazepam as needed E. Prescribe mirtazapine Question 81 A 68-year-old woman presents with left-sided weakness and dysarthria 2 hours after symptom onset. A CT scan confirms an ischaemic stroke with no evidence of haemorrhage. She has no contraindications to anticoagulation. What is the next most appropriate step in her management? A. Alteplase B. Aspirin C. Refer for emergency thrombectomy D. Statin E. Clopidogrel Question 82 A 32-year-old woman has had two episodes of optic neuritis and one episode of leg weakness over the past 3 years. Neurological examination reveals hyperreflexia. What is the gold standard investigation for confirming the diagnosis? A. Visual evoked potentials B. CT brainC. MRI brain and spinal cord D. Lumbar puncture E. Autoimmune blood screen Question 83 A 75-year-old man who is bedbound following a stroke develops a non-blanching, erythematous area over his sacrum. The skin is intact, but there is mild warmth and tenderness. What is the most appropriate classification of this pressure sore? A. Grade 1 B. Grade 2 C. Grade 3 D. Grade 4 E. Unstageable Question 84 A 47-year-old presents to her GP because she has felt a lump in her left breast when showering. What are the components of a triple assessment? A. Clinical examination, fine-needle aspiration, serum hormone concentrations B. Mammography, sentinel lymph node excision, fine-needle aspiration C. Clinical examination, mammography, fine-needle aspiration D. Clinical examination, BRCA 1 and 2 genetic testing, mammography E. Fine-needle aspiration, Mammography, Bone scan Question 85 An 81-year-old female patient is being treated on an acute ward for an infective exacerbation of COPD. One night, she slips on the floor while mobilising to the toilet. Upon examination, she complains of severe pain in her left hip and is unable to weight bear. An X- ray of her pelvis and hip is performed, which reveals a fracture. What radiological finding would most support a diagnosis of Paget's disease? A) Neck of femur (NOF) fracture B) Osteopenia C) Looser zone D) Loss of joint space E) Bony enlargement of the pubic ramiQuestion 86 A 68-year-old man is brought to the emergency department with confusion, fever, and hypotension. His heart rate is 110 bpm, temperature 39.1°C, blood pressure 90/50 mmHg, and respiratory rate 26/min. On examination, he is drowsy and has reduced urine output. Which of the following investigations should be performed first as part of the initial management? A. Chest X-ray B. Arterial blood gas C. Blood cultures D. CT abdomen and pelvis E. Serum lactate Question 87 A 50-year-old man is found to have a raised white cell count during a routine check-up. He feels well, though has noticed some abdominal fullness. Blood tests show leukocytosis with increased basophils. Bone marrow aspirate shows a full range of myeloid cell maturation. What translocation would confirm the diagnosis? A. t(9;22) B. t(8;14) C. t(15;17) D. t(11;14) E. t(14;18) Question 88 A 47-year-old man with a background of alcoholism presents to A&E with severe chest pain, shortness of breath, and vomiting. Clinical exam reveals subcutaneous emphysema in the neck and upper chest. Bloods show raised inflammatory markers and CXR reveals mediastinal widening. What is the best initial investigation to confirm the diagnosis? A. Barium swallow B. Plain chest X-ray C. Contrast-enhanced CT chest D. OesophagogastroduodenoscopyE. 24-hour pH manometry Question 89 A 65-year-old lady presents with severe deep pain in the left eye and nausea over the last hour. She has no past medical history and wears glasses for hypermetropia. Her pulse is 115 bpm, blood pressure is 138/78 mmHg, and she is afebrile. Her left eye is red, with reduced vision and is semi-dilated, not reactive to light. Tonometry reveals a raised intraocular pressure in the left eye. The right eye is unaffected. What investigation would be useful to confirm the underlying diagnosis? A: Fluoroscein angiography B: Fundoscopy C: Slit lamp D: Gonioscopy E: Optical coherence tomography Question 90 A 24-year-old man is brought to hospital with confusion and generalised seizures. He had a headache and low-grade fever for 3 days before becoming confused. On exam, he has no rash or neck stiffness. Lumbar puncture is performed, showing lymphocytosis, normal glucose, and mildly raised protein. What is the most appropriate empirical treatment? A. IV ceftriaxone and IV vancomycin B. IV acyclovir and IV ceftriaxone C. IV acyclovir alone D. IV benzylpenicillin E. Dexamethasone and ceftriaxone Question 91 A 71-year-old woman presents with new-onset headache, scalp tenderness, and jaw claudication. ESR is 92 mm/h. She is started on treatment without delay. What is the most appropriate next investigation? A: MRI brain B: CT temporal arteryC: Temporal artery biopsy D: Carotid Doppler E: Serum ACE level Question 92 A 55-year-old man with a history of heavy alcohol use is admitted with suspected acute pancreatitis. His amylase is 780 U/L (normal <100), and he is haemodynamically stable. After 48 hours, he develops rising creatinine, hypoxaemia, and fluctuating consciousness. What pathophysiological process is most responsible for these new findings? A. Hypocalcaemia due to fat necrosis B. Systemic inflammatory response syndrome (SIRS) and multiorgan dysfunction C. Acute cholecystitis with ascending cholangitis D. Hypovolaemia-induced pre-renal AKI E. Pancreatic pseudocyst formation Question 93 A 45-year-old woman presents to the GP with a 3-month history of intermittent vertigo, tinnitus, and hearing loss in her right ear. The episodes of vertigo last for 20-30 minutes and occur several times a week. There is no history of trauma or upper respiratory infection. Based on her symptoms, what is the most likely diagnosis? A. Benign paroxysmal positional vertigo (BPPV) B. Meniere’s disease C. Acoustic neuroma D. Labyrinthitis E. Vestibular neuritis Question 94 A 17-year-old boy sustains a minor cut while shaving. What is the first step in the formation of a temporary platelet plug at the site of endothelial injury? A. Platelet aggregation via GpIIb/IIIa receptors B. Platelet adhesion via Gp1b receptors C. Vascular spasm and vasoconstriction D. Activation of fibrinogen E. Release of thromboxane A2 and ADPQuestion 95 A 45-year-old man with known Addison’s disease is brought to A&E with confusion, vomiting, and abdominal pain. He is hypotensive (BP 80/50 mmHg), tachycardic (HR 110 bpm), and appears dehydrated. His last hydrocortisone dose was missed due to a viral illness. Blood tests show hyponatraemia and hyperkalaemia. What is the most appropriate immediate management? A. Oral hydrocortisone and IV fluids B. IV hydrocortisone and IV fluids C. Fludrocortisone and oral glucose D. Dexamethasone and broad-spectrum antibiotics E. IV saline and antiemetics Question 96 A 19-year-old man presents to ED at night with severe left scrotal pain. He describes the pain as coming on suddenly; it is severe and ongoing for the last four hours, despite the patient having taken ibuprofen. He has vomited three times since the onset of the pain and has some nonspecific abdominal pain. The patient is a university student and plays for his university rugby team. He is sexually active, but has never had a sexually transmitted infection and denies any unprotected sexual encounters in the last six months. On examination, the scrotal sac is swollen and appears erythematous. When stroking the inner thigh, the right testicle retracts, however the left testicle remains in place. Elevation of the scrotal sac does not relieve the patient’s pain. What is the most appropriate initial management? A. Scrotal USS B. Prescribe oral antibiotics C. Emergency surgical exploration +/- orchiopexy D. Refer to the urology day team for a review E. Reassure and discharge Question 97 A 67-year-old man presents to the respiratory clinic with a 6-month history of progressively worsening shortness of breath and a dry, non-productive cough. He has a 40-pack-year smoking history and works as a carpenter. On examination, he has fine crackles on auscultation at both lung bases and no signs of clubbing. A high-resolution CT scan reveals bibasilar reticular opacities with honeycombing. Which of the following pulmonary function test findings is most consistent with the diagnosis in this patient?A. Decreased FVC, normal FEV1/FVC ratio, and decreased DLCO B. Decreased FVC, normal or increased FEV1/FVC ratio, and normal DLCO C. Decreased FEV1, decreased FVC, and increased FEV1/FVC ratio D. Normal FVC, normal FEV1/FVC ratio, and decreased DLCO E. Increased FVC, decreased FEV1/FVC ratio, and decreased DLCO Question 98 A 60-year-old woman presents with acute abdominal pain and vomiting. She had a total abdominal hysterectomy 6 days ago. An Abdominal X-ray is performed: What is the most likely diagnosis? A. Paralytic ileus B. Bowel perforation C. Adhesive small bowel obstruction D. Ascites E. Sigmoid volvulus Question 99 An 84-year-old woman with known osteoarthritis is admitted following a fall. She is alert but very withdrawn, avoids eye contact, and is slow to respond. She is eating poorly and often appears to be "in her own world." Nursing staff report that she was more interactive yesterday. Her observations and blood results are within normal limits.What is the most likely cause of her current condition? A. Early dementia B. Depressive episode C. Hypoactive delirium D. Psychotic episode E. Frontal lobe stroke Question 100 A 44 M has been visiting his general practice (GP) frequently due to persistently high blood pressure at 155/95 mmHg resistant to anti-hypertensive medications. He has a PMHx of recurrent UTI and FHx of renal conditions diagnosed at a young age. His father and grandfather have had heart problems and died from a stroke. His GP thus arranged for an ultrasound scan of the abdomen and urinary tract which revealed multiple cysts on both kidneys. What is the most common extrarenal manifestation of this condition? A. Stroke B. Aortic dissection C. Berry aneurysms D. Mitral valve prolapse E. Liver cysts Question 101 A 35-year-old woman presents with a 3-month history of itchy, scaly plaques on her elbows and scalp. She has a background of anxiety and recently had a throat infection. On examination, there are well-demarcated, erythematous plaques with silvery scales on her extensor surfaces and scalp. Which of the following is most characteristic of this condition? A. Koebner phenomenon and Auspitz sign B. Central clearing and active edge C. Widespread nodules D. Target lesions with dusky centers E. Pseudopodia at the advancing margin Question 102 A 72-year-old woman was admitted to hospital 5 days ago for management of a hip fracture. She now develops a fever of 38.6°C, productive cough with green sputum, andshortness of breath. On examination, she has coarse crackles in the right lower zone. Her oxygen saturation is 90% on room air. Which of the following organisms is the most likely cause of her current condition? A. Streptococcus pneumoniae B. Mycoplasma pneumoniae C. Klebsiella pneumoniae D. Pseudomonas aeruginosa E. Staphylococcus aureus Question 103 A 68-year-old male presents with intermittent diarrhoea and weight loss over the past 3 months. He has also noticed some blood mixed in with the stool. On examination, a hard mass is felt in the left lower quadrant, and a digital rectal examination reveals a palpable, irregular rectal mass. A colonoscopy with biopsy confirms the presence of a rectal tumour. Which of the following investigations is most appropriate for staging the tumour? A. Abdominal X-ray B. Barium enema C. CT scan of the chest, abdomen, and pelvis D. Endoscopic ultrasound E. PET scan Question 104 A 17-year-old girl presents to her GP with a two week history of headaches. She notices that they come on in the morning that improves after she gets out of bed and she has vomited twice in the past few weeks. She had not noticed any concordant vision changes or changes in sensation. She doesn’t have a fever, muscle or joint pains or any coryzal symptoms. Her past medical history includes PCOS. She has a BMI of 31. The GP checks her eyes with a fundoscope. What would they expect to see? A. Pale fundus with a red dot in the centre B. Blurring of the optic disc C. Yellow plaques D. AV nipping E. Blot haemorrhagesQuestion 105 A 35-year-old man presents with progressive muscle weakness, fasciculations, and hyperreflexia. There is no sensory involvement. What is the most likely diagnosis? A. Multiple sclerosis B. Myasthenia gravis C. Motor neuron disease (ALS) D. Guillain-Barré syndrome E. Peripheral neuropathy Question 106 A 50-year-old man presents with chronic sinusitis, haematuria, and haemoptysis. He has a positive c-ANCA (PR3-ANCA). What is the most likely diagnosis? A. Microscopic polyangiitis B. Goodpasture’s syndrome C. Granulomatosis with polyangiitis (Wegener’s) D. Eosinophilic granulomatosis with polyangiitis E. SLE Question 107 A 52-year-old man presents with post-prandial bloating, regurgitation of undigested food, and a sensation of a lump in the throat. He does not experience heartburn or nausea. Halitosis is noted on examination. What is the most likely diagnosis? A. Achalasia B. Zenker’s diverticulum C. GORD D. Gastritis E. Mallory-Weiss tear Question 108 A 17-year-old boy brought in to A&E after losing consciousness whilst out with his parents. He is shaking continuously and has been doing so for over an hour by the time he is seen bya doctor. His parents mention that he has had a previous seizure but that terminated by itself. Given the most likely diagnosis, what is the most appropriate step for this patient? A. MRI B. IV phenytoin C. Oral sodium valproate D. CT head E. IV lorazepam Question 109 A 50-year-old man is found to have a raised white cell count during a routine check-up. He feels well, though has noticed some abdominal fullness. Blood tests show leukocytosis with increased basophils. Bone marrow aspirate shows a full range of myeloid cell maturation. What translocation would confirm the diagnosis? A. t(9;22) B. t(8;14) C. t(15;17) D. t(11;14) E. t(14;18) Question 110 A 65-year-old male with a 40-pack-year smoking history presents to the clinic with increasing shortness of breath, wheezing, and chronic cough productive of sputum for the past 6 months. On examination, he has a prolonged expiratory phase, decreased breath sounds, and wheezing. Which of the following is the most appropriate next step in investigating this patient's condition? A. High-resolution CT scan of the chest B. Arterial blood gas analysis C. Spirometry (FEV1/FVC ratio) D. Chest X-ray E. Sputum culture for bacterial pathogens Question 111 A 56-year-old Hispanic woman presents to the emergency department with a fracture of her distal radius. This is her second fracture in the last six months. She has a past medical historyof poorly controlled asthma and chronic kidney disease (CKD) stage 3. Specific blood tests are shown below: • Calcium: 2.25 mmol/L (normal range: 2.2 - 2.6) • Phosphate: 1.2 mmol/L (normal range: 0.8 - 1.5) • Alkaline Phosphatase (ALP): 78 IU/L (normal range: 25 - 115) • Parathyroid Hormone (PTH): 5.5 pmol/L (normal range: 1.6 - 8.5) • Vitamin D: 95 nmol/L (normal range: >50) What is the most likely cause of these repeated fractures? A) Paget's disease B) Multiple myeloma C) Osteoporosis D) Primary hyperparathyroidism E) Bone mineral deficiency secondary to chronic kidney disease Question 112 A 62-year-old woman has been diagnosed with oestrogen-receptor positive, HER-receptor negative breast cancer with a tumour measuring 4.5cm. Her last menstrual period was 7 years ago. She has undergone a total mastectomy and axillary node clearance. Histology shows clear margins and no axillary lymph node involvement. What adjuvant treatment would likely be a part of her management? A. Anastrozole B. Tamoxifen C. No additional treatment D. Radiotherapy E. Trastuzumab Question 113 A 3-year-old patient is brought in by his mother who reports a painless swelling in his neck. The mother says that it appeared gradually and is afraid it may be something sinister. On examination, the lump appears to be in the midline and moves up on both tongue protrusion and swallowing. The mother reports no other symptoms. Given the most likely diagnosis, what is the next most appropriate step? A. Immediate surgical referral B. Ultrasound C. Antibiotics D. Reassure the patient and advise to closely observe E. Serum CalciumQuestion 114 A 45-year-old man with a history of psoriasis develops joint pain and swelling, especially in the DIP joints. He also reports morning stiffness lasting over 30 minutes. On exam, there is nail pitting and dactylitis. What is the most likely diagnosis? A: Rheumatoid arthritis B: Osteoarthritis C: Gout D: Psoriatic arthritis E: Septic arthritis Question 115 A patient with medullary thyroid cancer is found to have mucosal neuromas and marfanoid body habitus. Genetic testing reveals a RET mutation. Which screening test should be urgently performed? A. Serum prolactin B. Plasma metanephrines C. Colonoscopy D. OGTT E. 24-hour urinary 5-HIAA Question 116 A 77-year-old woman patient presents to the Emergency Department following a fall. Her past medical history includes hypertension and type 2 diabetes. She is a smoker with a BMI of 34 kg/m². Her only family history is high cholesterol in both her father and older sister, who both died of a heart attack. She denies any head trauma following the fall. Her pulse is 78bpm and regular. After a full neurological examination, you find her left arm and left leg power 3/5. You also note her smile is asymmetrical and droops on the left side. What is the most likely underlying aetiology causing her symptoms? A: Atonic seizure attack B: Atrial appendage dislodged by atrial fibrillation C: Emboli caused by atherosclerosis D: Intracerebral haemorrhage E: Postural hypotensionQuestion 117 A 40-year-old man attends his GP with home BP readings averaging 172/104 mmHg over two weeks. He has no symptoms or known medical history. Which is the most common secondary cause of hypertension? A: Hypertensive encephalopathy B: Renal disease C: Coarctation of the aorta D: Cushing’s syndrome E: Hypothyroidism Question 118 Upon examination with the swinging torch test, both of the patient’s pupils dilate when light is shone into the left eye, but both constrict when shone into the right eye. Where is the site of the lesion? A: Right retina or optic nerve B: Left retina or optic nerve C: CN III D: CN IV E: Sympathetic nerve fibres Question 119 Which of the following diabetes medications is associated with weight loss and reduced cardiovascular mortality? A. Gliclazide B. Sitagliptin C. Metformin D. Empagliflozin E. Insulin glargine Question 120 A 30-year-old man presents to ED with severe right-sided flank pain. It started two days ago as a colicky pain but has now become constant. It is associated with vomiting and rigour. A recent set of observations reveals a fever of 39.4℃ and BP of 98/78. A CT-KUB is performed which shows a 3cm stone in the right ureter and an enlarged right kidney. What is the most appropriate management in order to reduce the immediate risk of kidneyinjury? A. Extracorporeal shockwave lithotripsy B. IV fluids and PR diclofenac C. Percutaneous nephrostomy D. Ureteroscopy E. IV broad-spectrum antibiotics Question 121 A 4-year-old boy with known haemophilia A presents with a swollen and painful right knee after a minor fall. What is the most appropriate next step in management? A. Platelet transfusion B. Desmopressin (DDAVP) C. Recombinant factor VIII D. Fresh frozen plasma E. Intravenous immunoglobulin (IVIG) Question 122 A 58-year-old man presents with progressive shortness of breath, abdominal distension, and bilateral ankle swelling over 6 months. He has no significant past medical history. On examination, he has raised JVP with a positive Kussmaul’s sign, hepatomegaly, and shifting dullness. ECG shows low voltage QRS complexes. CXR shows no pulmonary congestion. You suspect a cardiac cause. What is the most appropriate initial investigation? A. Cardiac MRI B. Abdominal ultrasound C. Echocardiography D. Endomyocardial biopsy E. BNP Question 123 A 40-year-old woman presents with dry eyes, dry mouth, and arthralgia. Schirmer’s test is positive. ANA and anti-Ro are positive. What is the most likely diagnosis?A. Rheumatoid arthritis B. Sjögren’s syndrome C. Systemic sclerosis D. Polymyositis E. Sarcoidosis Question 124 A 50-year-old woman collapses at home. Paramedics find her to be hypotensive (BP 80/40), tachycardic (HR 120), and severely hypoxic. She has a history of recent surgery and swollen left leg. In A&E, bedside echocardiography shows a dilated right ventricle. What is the most appropriate next step in management? A. Start heparin infusion B. Immediate thrombolysis C. Arrange urgent CT pulmonary angiogram D. Give oral rivaroxaban E. Start oxygen and await D-dimer Question 125 A 45-year-old woman presents with bilateral temporal headaches, jaw claudication, and scalp tenderness. ESR is 85 mm/hr. What is the first-line treatment according to NICE? A. Ibuprofen B. Paracetamol C. Low-dose aspirin D. Oral prednisolone 40–60 mg E. Methotrexate Question 126 A 65-year-old man presents with fatigue, reduced urine output, and mild lower limb swelling. He was recently started on ramipril for hypertension and takes long-term furosemide. Blood pressure is 102/62 mmHg, and JVP is low. Bloods show: • · Urea 24 mmol/L (normal range: 2.0 – 7 mmol/L) • · Creatinine 310 µmol/L (baseline 105) (normal range: 55 – 120 umol/L) • · K⁺ 5.7 mmol/L (normal range: 3.5 – 5.0 mmol/L) Urinalysis is bland (no protein or blood), and renal ultrasound is normal.What is the most likely cause of his AKI? A. Acute interstitial nephritis B. Acute tubular necrosis C. Pre-renal AKI due to ACE inhibitor + diuretic D. Crescentic glomerulonephritis E. Obstructive uropathy Question 127 A 60-year-old male presents with abdominal pain in the right upper quadrant for the past 2 months and also reports his clothes starting to feel more loose. His wife has noticed his eyes starting to look a bit more yellow. He has a past medical history of chronic hepatitis C and cirrhosis. On examination, there is a non-tender mass palpated in the right upper quadrant of the abdomen. His serum alpha-fetoprotein (AFP) level is elevated. What is the most likely diagnosis? A. Hepatocellular carcinoma B. Gastric cancer C. Pancreatic Cancer D. Gallbladder carcinoma E. Cholangiocarcinoma Question 128 A 34-year-old man presents with acute monoarthritis of the right knee. Joint aspiration shows needle-shaped negatively birefringent crystals under polarised light microscopy. What is the most likely diagnosis? A: Pseudogout B: Septic arthritis C: Osteoarthritis D: Gout E: Rheumatoid arthritisQuestion 129 A 22-year-old woman with known type 1 diabetes presents to A&E with nausea, vomiting, and deep rapid breathing. Her blood glucose is 24 mmol/L, pH is 7.1, ketones are 5.2 mmol/L, and potassium is 5.4 mmol/L. What is the most appropriate next step in her management? A. IV bicarbonate B. IV insulin immediately C. IV 0.9% saline and reassess potassium D. Oral glucose and fluids E. IM glucagon Question 130 A 12-year-old girl presents to the emergency department with facial and eye pain. She has recently had a sinus infection. On examination there is proptosis of the left eye, with ophthalmoplegia. The palpebra is red and swollen. Her temperature is 39.5 ºC and she looks unwell. Given the most likely diagnosis, what is the immediate management plan? A: Urgent contrast CT head B: Urgent non-contrast CT head C: Admit into hospital with oral antibiotics D: Admit into hospital with IV antibiotics E: Discharge with oral antibiotics Question 131 A 63-year-old man attends his GP for a routine check-up. He feels well with no symptoms. On examination, nothing abnormal is found. His lipid profile shows: Total cholesterol: 6.3 mmol/L LDL: 4.2 mmol/L HDL: 0.7 mmol/L No history of cardiovascular disease or diabetes. After 4 months of lifestyle changes, his lipid profile remains largely unchanged. What is the most appropriate next step in managing this patient? A: Prescribe ezetimibe B: Prescribe 10 mg atorvastatin C: Prescribe 20 mg atorvastatin D: Prescribe 80 mg atorvastatin E: Watchful waitingQuestion 132 A 34-year-old man with a history of mild hypertension is found during a routine review to have bilateral corneal arcus and xanthomata over his Achilles tendons. He is asymptomatic. His family history is unknown due to adoption. His GP suspects familial primary hyperlipidaemia and orders a lipid panel. Which of the following total cholesterol results would warrant further investigation for familial primary hyperlipidaemia? A: 3.7 mmol/L B: 5.8 mmol/L C: 4.9 mmol/L D: 6.9 mmol/L E: 7.7 mmol/L Question 133 A 62-year-old woman presents to the urgent care clinic with sudden-onset facial and tongue swelling. She has no rash, itchiness, or difficulty breathing. Her past medical history includes hypertension, type 2 diabetes, and ischaemic heart disease. She was recently started on a new medication by her GP, though she does not remember the name. Which medication is most likely responsible for her symptoms? A: Ramipril B: Spironolactone C: Nifedipine D: Atorvastatin E: Bisoprolol Question 134 An ECG is performed on a patient in the cardiology ward. On the ECG there are regular p waves present, and a QRS complex is associated with each p wave. The PR interval is 0.26 seconds. There are no missed p waves. What is the most likely diagnosis? A: 1st degree heart block B: 2nd degree heart block - Mobitz type I C: 2nd degree heart block - Mobitz type IID: 3rd degree heart block E: Sinus rhythm Question 135 A 42-year-old male patient presents to your GP clinic with a background of coughing, shortness of breath, and fever. The patient is generally fatigued and has a headache. He is known to be HIV positive. On examination, you notice dullness to percussion in the middle zone of the right lung. Which of the following is the most likely causative organism? A. Staphylococcus aureus B. Pneumocystis jirovecii C. Streptococcus pneumoniae D. Legionella pneumophila E. Haemophilus influenzae Question 136 A 24-year-old man is found lying on the pavement at the side of the road. Paramedics note he is barely conscious, breathing very slowly, and has pinpoint pupils. They also note track marks on his left arm. What is the next most appropriate step for management? A. Intravenous thiamine B. CPR C. Oral naltrexone D. Intravenous naloxone E. Intubation and ventilation Question 137 A 26-year-old presents with 3 months of intermittent RLQ crampy pain, non-bloody diarrhoea, and 5 kg weight loss. He has already failed a 4-week tapering course of oral prednisone. Exam shows a tender perianal skin tag and a small external fistula opening. Colonoscopy confirmed patchy ileocolonic ulcers. What is the best next step in management? A. Start azathioprine B. Initiate infliximab C. Perform seton placement alone D. Switch to methotrexate E. Start sulfasalazine Question 138 A 58-year-old man is brought to A&E by paramedics. He smells strongly of alcohol and is reported to be repeatedly asking the staff for a drink. Over time, the man is observed to have tremors, sweating, and anxiety. What is the next most appropriate step for management? A. Diazepam B. Chlordiazepoxide C. Carbamazepine D. Phenytoin E. Supportive care Question 139 A 34-year-old with chronic steatorrhea, iron-deficiency anaemia, and mild peripheral neuropathy has positive anti–tissue transglutaminase IgA. Duodenal biopsy shows villous blunting. What is the most appropriate first-line therapy? A. Low-FODMAP diet B. Gluten-free diet C. Pancreatic enzyme replacement D. Lactose-free diet E. Elemental (amino acid) diet Question 140 A 25 year-old female visits her GP due to blood in her urine. She describes a discomfort and a burning sensation during urination and the urine is also described to have a foul smell. She has no other significant symptoms other than a mild fever. She is 40 weeks pregnant. What is the most appropriate treatment? A. Nitrofurantoin 7 days B. Nitrofurantoin 3 days C. Amoxicillin 7 daysD. Trimethoprim 3 days E. Trimethoprim 7 days Question 141 A 70-year-old male presents to his GP following abnormal blood test results. He has a past medical history of recently diagnosed small cell lung cancer, hypertension, and hyperlipidaemia. He is currently undergoing chemotherapy and is taking simvastatin, amlodipine, and hydrochlorothiazide. The patient reports feeling generally well but has noticed increasing fatigue over the past week. His examination is unremarkable, and his vital signs are stable. He is for full resuscitation. Blood test results are: • Sodium: 138 mmol/L (normal range: 135 - 145) • Potassium: 4.3 mmol/L (normal range: 3.5 - 5.3) • Creatinine: 105 µmol/L (normal range: 60 - 120) • Calcium: 2.9 mmol/L (normal range: 2.2 - 2.6) What is the most appropriate management? A) Oral alendronate B) Encourage oral fluid intake C) Intravenous rehydration D) Hold Hydrochlorothiazide and encourage oral fluid intake E) Refer for outpatient denosumab infusion Question 142 A 30-year-old man collapses in a restaurant shortly after eating dishes contaminated with peanuts. He has a widespread urticarial rash, audible wheeze, and is hypotensive at 78/40 mmHg. An ambulance is called by the restaurant staff. What is the next most appropriate step for management? A. Administer IV adrenaline B. Administer IM adrenaline C. Lie the patient flat D. Give high-flow oxygen E. Give non-sedating antihistaminesQuestion 143 A 6-year-old boy presents with 12 hours of diffuse, mild abdominal pain centred in the RLQ, low-grade fever, and a preceding viral upper-respiratory illness. Ultrasound shows multiple enlarged mesenteric lymph nodes but a normal appendix. WBC is mildly elevated. What is the best management? A. Appendectomy B. IV broad-spectrum antibiotics C. Conservative observation with hydration and NSAIDs D. CT-guided lymph node biopsy E. Repeat ultrasound in 24 hours Question 144 A 72-year-old man presents with excruciating abdominal pain, out of proportion to examination findings. He has a background of AF, CKD stage 3, and recent NSTEMI. Initial bloods show raised WCC and lactate. What is the most definitive next investigation to confirm the diagnosis? A. Abdominal ultrasound with Doppler B. Erect CXR C. Mesenteric angiography via CT D. Plain AXR E. Diagnostic laparoscopy Question 145 A 29-year-old woman reports 6 months of recurrent, crampy lower-abdominal pain relieved by defecation, bloating, and alternating constipation-diarrhoea. Physical exam and labs (CBC, CRP, TSH, electrolytes) are all normal. What is the most appropriate next step? A. Colonoscopy B. Trial of a low-FODMAP diet and fibre supplementation C. Empiric antibiotics (rifaximin) D. Abdominal CT E. Prescribe an antispasmodic (e.g., hyoscine)Question 146 A 48-year-old man experiencing homelessness presents to the GP with fatigue, persistent diarrhoea, and a rash on the backs of his hands and forearms that worsens in sunlight. On examination, he appears confused and disoriented. His diet is poor, and he reports drinking several cans of strong lager daily. What is the most likely diagnosis? A. Vitamin B1 deficiency B. Vitamin B3 deficiency C. Vitamin B6 deficiency D. Vitamin B9 deficiency E. Vitamin B12 deficiency Question 147 A 60-year-old man presents to his GP with a flushed complexion, headaches and itching which is especially worse after taking hot baths. On examination, he has a palpable mass in the left upper quadrant. The GP arranges urgent blood tests which reveal a raised haemoglobin (195), raised haematocrit (0.64) and a mild thrombophilia (490). Based on the suspected underlying cause, which gene is most likely to have a mutation? A. ABL B. ANK1 C. TET2 D. JAK2 E. BCR Question 148 A 67-year-old woman presents with a painful lump in the groin that is tender, irreducible, and located inferolateral to the pubic tubercle. She has nausea but no vomiting. There are no bowel sounds over the lump. What is the most likely complication if not treated promptly? A. Obstruction of the small bowel B. Incarceration leading to strangulation C. Peritonitis from appendix perforationD. Ovarian torsion E. Ureteric obstruction due to extrinsic compression Question 149 A 34-year-old man presents to A&E after being stung by a wasp. He is conscious but complains of no pain and no swelling at the site. He has no airway compromise or systemic symptoms. What is the most appropriate next step in management? A. Intramuscular adrenaline B. Intravenous antibiotics C. Apply a tourniquet above the sting site D. Remove the sting and apply cold compress E. Administer tetanus immunoglobulin Question 150 A 42-year-old man complains of severe, constant anal pain worse with sitting, plus fever. On exam you note a tender, fluctuant swelling just lateral to the anal verge. What is the most appropriate immediate management? A. High-fibre diet and sitz baths B. Oral antibiotics and outpatient follow-up C. Urgent incision and drainage D. MRI pelvis E. Perianal ultrasound for mappingMark Scheme: Question 1 Answer: B. Start furosemide Explanation: This patient has classic symptoms and signs of congestive heart failure with reduced ejection fraction (HFrEF), including orthopnoea, paroxysmal nocturnal dyspnoea, S3, and crackles. His EF is <40%, confirming HFrEF. While long-term management includes ACE inhibitors and beta-blockers (already likely on bisoprolol), the priority is symptom relief via diuresis, especially with signs of fluid overload. Furosemide, a loop diuretic, is the correct choice. Digoxin does not improve mortality and is not first-line. Isosorbide mononitrate and valve surgery are not indicated unless a clear underlying valvular pathology is identified. Question 2 Answer: C) Chronic limb ischaemia with intermittent claudication Explanation: This patient has classic intermittent claudication: exertional calf pain relieved by rest, with signs of arterial insufficiency and ABPI <0.9. ABPI of 0.55 indicates moderate PAD. There is no rest pain or tissue loss, so this does not meet criteria for critical limb ischaemia. Acute ischaemia would be sudden with the 6 Ps. Venous disease causes swelling and pigmentation, not claudication. Diabetic neuropathy causes numbness, not exertional pain. Question 3 Answer: B. Refer urgently for surgical debridement Explanation: This presentation is highly suggestive of necrotising fasciitis, a rapidly progressive soft tissue infection requiring emergency surgical debridement and IV antibiotics. Pain out of proportion, systemic signs, and skin discoloration with blistering are red flags. Oral antibiotics are insufficient, and Doppler is inappropriate unless DVT is suspected without infection signs. Question 4 Answer: D. Oesophageal adenocarcinoma Explanation: This patient has presented with a typical history of progressive dysphagia (difficulty swallowing) accompanied by unintentional weight loss, raising strong suspicions of oesophageal cancer. Progressive dysphagia occurs due to the growing size of the tumour, which gradually narrows the oesophageal lumen. GORD is a key risk factor associated with oesophageal adenocarcinoma, whereas oesophageal squamous cell carcinoma is morecommonly associated with smoking and alcohol. Barrett’s oesophagus results from chronic acid reflux and is a premalignant condition, associated with metaplasia (change in epithelium cell type). Whilst Barrett’s oesophagus and peptic strictures are suitable differential diagnoses, they are not usually associated with weight loss. Achalasia is a motility disorder caused by the failure of the lower oesophageal sphincter to relax. This also presents with dysphagia, however, patients typically have difficulty swallowing solids and liquids at the same time, rather than the progressive dysphagia of solids first and then liquids seen in this case. Question 5 Answer: C. Renal cell carcinoma The main differentials for frank haematuria are cancer, stones and infection. The lack of pain means urolithiasis is less likely. The lack of other STI symptoms plus the other symptoms the patient is experiencing make it less likely. The lack of changes to urinary habits and lack of fever means lower UTI are less likely. The lack of haemodynamic instability and signs of sepsis indicate pyelonephritis is less likely. The smoking history, weight loss, fatigue, ballotable mass and renal angle tenderness are more in keeping with a malignancy. Question 6 Answer: C. Pituitary adenoma Explanation: The key clinical finding is bitemporal hemianopia, which suggests a lesion at the optic chiasm, where nasal retinal fibres (responsible for the temporal visual fields) decussate. The most common cause of a compressive lesion in this area is a pituitary adenoma, which arises from the sella turcica and can grow superiorly to compress the optic chiasm. Other options are less likely: Retinal detachment (A) typically causes monocular vision loss or a shadow over part of the visual field, not a chiasmal field defect. Multiple sclerosis (B) and optic neuritis (D) usually cause unilateral central vision loss with pain on eye movement. Normal pressure hydrocephalus (E) can cause cognitive decline, gait disturbance, and urinary incontinence but does not typically affect vision in this pattern. Question 7 Answer: E. Transthoracic echocardiography Explanation: This is a classic presentation of hypertrophic obstructive cardiomyopathy (HOCM), a leading cause of sudden cardiac death in young athletes. It often has a familial (autosomal dominant) inheritance. Echo is the most sensitive and specific tool for detecting ventricular hypertrophy and outflow obstruction. ECG may show changes like LVH or deep Q waves but is not diagnostic. Chest X-ray and CT angiogram are unhelpful in structural cardiomyopathies. TOE (Transoesophageal echo) provides more detailed images of posterior cardiac structures but is invasive and not first-line for diagnosing HOCM. It is mainly reserved for cases where TTE is inconclusive (e.g. in poor acoustic windows), or for valvular assessment or endocarditis, not routine screening of cardiomyopathies. TTE, in contrast, isnon-invasive, widely available, and sufficient to make the diagnosis in the majority of HOCM cases. Question 8 Answer: A. Gallstones Explanation: The history of severe colicky right upper quadrant pain worse after eating fatty food is classic for biliary colic due to gallstones. LFTs show a cholestatic pattern, indicating a post-hepatic cause of jaundice: High ALP (430 IU/L) is indicative of biliary obstruction. Moderate rise in bilirubin. Mild transaminitis (AST, ALT) can happen secondary to obstruction. This biochemical profile fits with obstructive jaundice, often from gallstones in the common bile duct. (B) Alcoholic hepatitis would show AST:ALT ratio >2, ALP is not usually elevated and no history of alcohol misuse. (C) Viral hepatitis would show markedly elevated AST and ALT (often in the hundreds to thousands), not just mildly raised. ALP may be mildly raised or normal. No mention of viral prodrome or risk factors e.g. IV drug use. (D) Hepatocellular carcinoma would present with constitutional symptoms e.g. weight loss. (E) Gilbert’s syndrome would cause isolated unconjugated hyperbilirubinemia with no elevation of ALP, AST or ALT. Question 9 Answer: B Sarcoidosis Explanation: This presentation is classic for sarcoidosis – bilateral hilar lymphadenopathy, erythema nodosum and arthralgia. TB cause bilateral lymphadenopathy, but is likely to be associated with constitutional symptoms and productive cough. Hodgkin lymphoma will present with B symptoms and painless lymphadenopathy. SLE will typically present with malar rash and not nodular rash. Question 10 Answer: C Supportive care with fluids and rest Explanation: The patient has classic symptoms of infectious mononucleosis which is caused by Ebstein-Barr virus and is typically self limiting. Mainstay of treatment is supportive care – hydration, rest, analgesics/ antipyretics. Oral corticosteroids are not first-line but may be considered if there is upper airway obstruction, severe tonsillar enlargement, or complications such as haemolytic anaemia. Antiviral therapy with acyclovir has limited benefit in uncomplicated EBV infection and is not routinely recommended. Oral amoxicillin should be avoided in EBV because it commonly causes a maculopapular rash in patients with mononucleosis. Question 11 Answer: B. Primary adrenal insufficiency (Addison’s disease) Explanation: This patient has classical signs of Addison’s disease, including fatigue, weight loss, hypotension, hyperpigmentation, and electrolyte abnormalities (hyponatraemia andhyperkalaemia). The high ACTH and low cortisol confirm primary adrenal failure. In contrast, secondary adrenal insufficiency (from pituitary disease) would show low ACTH and no hyperkalaemia (as aldosterone is preserved). Cushing’s would present with weight gain and hypertension. SIADH causes hyponatraemia but not hyperkalaemia. Phaeochromocytoma would present with hypertension and episodes of palpitations, not chronic fatigue. Question 12 Answer: D Explanation: Given that this patient has had an epidural a day ago and is now presenting with a headache and nausea, post-dural tap is the most likely answer. Intracerebral haemorrhage during pregnancy or puerperium is one of the leading causes of maternal deaths worldwide, but is a rare event and more common in patients with existing pregnancy risk factors. You would expect a past medical history of headaches if this was a migraine or medication overuse headache. The severe nature of this headache and the pain originating from the occipital region makes a tension type headache less likely. Question 13 Answer: B Explanation: This patient is having a lower urinary tract infection as evidenced by the dysuria and increased frequency. A three-day course of nitrofurantoin is the most appropriate initial management of a UT in a female who isn’t pregnant. A seven-day course of nitrofurantoin is used to treat urinary tract infections in males and pregnant females. UTIs are more complicated in males and more high risk during pregnancy, hence the longer antibiotic course. Amoxicillin would be inappropriate in this patient as she is penicillin allergic. Gentamicin is a broad-spectrum antibiotic which wouldn’t typically be used to treat a simple UTI. There isn’t sufficient evidence for cranberry juice as a treatment for UTIs. Question 14 Answer: C Explanation: The patient has hypercalcaemia, low phosphate, and elevated PTH, which is classic for primary hyperparathyroidism (PHPT). In PHPT, excessive PTH increases calcium levels. Malignancy would suppress PTH. Tertiary hyperparathyroidism (E) occurs in chronic kidney disease, which is not present. Secondary hyperparathyroidism (D) presents with low or normal calcium, and pseudohypoparathyroidism (B) features low calcium but high PTH. Question 15 Answer: C Explanation: This patient most likely has glandular fever as she is young and presenting with a sore throat, cervical lymphadenopathy and enlarged tonsils. This is an infection caused by the Epstein-Barr virus, which is detected by a heterophile antibody test (Monospot). EBV serology is a second line test that is used after two negative Monospot tests. Severe EBV infection can cause deranged LFTs, however this is not indicated by the history. TheMantoux test is for TB. Throat swabs are not commonly used as bacterial and viral tonsillitis can be differentiated clinically using the CENTOR score. Question 16 Answer: D. Plasma exchange Explanation: This patient has classic symptoms and signs of Thrombotic Thrombocytopenic Purpura (MAHA - schistocytes, Thrombocytopenia, AKI – raised urea, fever and neuro signs – confusion). The treatment for TTP is plasma exchange +/- corticosteroids. Question 17 Answer: D. Ischaemic stroke Explanation: The patient is most likely to have experienced a stroke given the sudden onset of their symptoms. The patient has forehead sparing suggesting a central lesion rather than Bell’s palsy. The pupil is dilated suggesting cranial nerve III involvement. The patient also has vascular risk factors such as hypertension and smoking. Temporal arteritis would present with symptoms like headaches, scalp tenderness and visual symptoms. Multiple sclerosis is rare to present early. Ramsay Hunt syndrome would present with pain and facial weakness as well as a rash around the ear. Question 18 Answer: C. Venous duplex ultrasound Explanation: This is a high-probability DVT (Wells ≥2). In such patients, the next step is compression (duplex) ultrasound of the affected leg. D-dimer is not needed in high-risk patients. CT or MR venography is rarely first-line. CT angiogram is used for suspected PE, not limb DVT. Question 19 Answer: D. Intranasal steroids Explanation: This patient has chronic rhinosinusitis given the symptoms and their duration (>12 weeks is considered chronic). Given this, first line would be intranasal steroids. Nasal irrigation is an adjunct that is recommended but the treatment option most appropriate would still be the steroids. Whilst longer durations of rhinosinusitis would hint at a bacterial cause, the clear nasal discharge and lack of systemic symptoms suggests that antibiotics are not indicated. Anterior rhinoscopy could be diagnostic of polyps which could be a cause of chronic rhinosinusitis but is not a treatment itself. Question 20Answer: C. Myelofibrosis Explanation: The hallmark of myelofibrosis is a fibrotic bone marrow leading to a dry tap on aspiration, often with massive splenomegaly (causing early satiety) and pancytopenia. Tear drop cells (dacrocytes) may also be seen. This distinguishes it from other myeloproliferative disorders. Question 21 Answer: B. Parkinson’s disease Explanation: Parkinson’s disease is a progressive neurodegenerative disorder characterised by loss of dopaminergic neurons in the substantia nigra. The classic triad includes resting tremor (typically "pill-rolling"), bradykinesia, and rigidity. Symptoms often start asymmetrically and improve with voluntary movement, which helps differentiate it from essential tremor. Essential tremor is typically a postural or action tremor with no associated rigidity or bradykinesia. Multiple system atrophy and progressive supranuclear palsy may have Parkinsonian features but usually include additional signs such as autonomic dysfunction or vertical gaze palsy, respectively. Huntington’s disease presents with chorea and cognitive decline, not tremor. Question 22 Answer: E. Chest x-ray Explanation: This symptoms (mild shortness of breath, low-grade fever, non-productive cough, and unintentional weight loss) are suggestive of pulmonary tuberculosis, especially considering his recent time in India, where TB is endemic. The most crucial investigation for making the diagnosis at this stage is a chest X-ray. It is a readily available, non-invasive, and essential first-line test in suspected TB, often revealing classic features such as upper lobe infiltrates, cavitation, or a miliary pattern. While further tests like sputum microscopy or cultures are required for microbiological confirmation, the chest X-ray is key in guiding the initial diagnosis and management. Other options such as CT chest or HIV testing may be helpful later, but they are not immediately necessary for making the diagnosis. Question 23 Answer: E Explanation: This man is presenting with encephalitis rather than meningitis, as evidenced by the personality changes that his girlfriend has noticed and lack of meningism symptoms (neck stiffness, photophobia). This rules out C. neoformans, N. meningitidis and M. tuberculosis, which tend to cause meningitis rather than encephalitis. T. gondi is a causative agent of encephalitis, however it tends to only affect immunocompromised populations. Therefore, in this patient, HSV is the most likely causative organism. Question 24Answer: D Explanation: This patient has severe hypocalcemia (1.6 mmol/L) with the classic Chvostek's sign, which suggests hypoparathyroidism following thyroidectomy. The best initial treatment is IV calcium gluconate to rapidly correct the hypocalcaemia and prevent complications like cardiac arrhythmias. Oral calcium carbonate (B) is used for chronic management but is not sufficient in acute symptomatic hypocalcaemia. High-dose vitamin D (A) is useful in long-term therapy but does not act fast enough. Loop diuretics (C) worsen hypocalcaemia by increasing calcium excretion. Thiazide diuretics (E) can increase calcium levels but are not first-line in acute settings. Question 25 Answer: B. Bag of worms With the history, there is a strong indication of malignancy. A renal carcinoma as indicated by the mass on the side of the abdomen, can compress the renal vein (usually left-sided), causing a back pressure on the testicular vessels. This a varicocele which is characterised by swollen tortuous veins which are known to feel like a ‘bag of worms’. Whilst it is possible to have a normal testicle, this would not indicate a renal cell carcinoma. Loss of the cremasteric reflex indicates testicular torsion. Swelling and tenderness eased by lifting up the testes indicates acute epididymo-orchitis. A soft transluminescant fluctuant lump indicates a hydrocoele which can be a sign of testicular cancer, however, the history suggest a renal cell carcinoma due to the location of the palpable mass. Question 26 Answer: E. Oesophagogastroduodenoscopy (OGD) with biopsy Explanation: This patient has presented with symptoms of an upper gastrointestinal malignancy. The epigastric pain, unintentional weight loss with a history of chronic H pylori infection raises suspicion of gastric cancer. This is further supported by the anaemia and Virchow’s node (lymphadenopathy in the left supraclavicular fossa). Also, melaena is black tarry stool which occurs following an upper GI bleed which in this case can be secondary to gastric cancer. As the blood moves along the digestive tract, it is digested becoming black and tarry. The gold standard investigation for diagnosis of gastric cancer is an oesophagogastroduodenoscopy (OGD) with a biopsy. CT abdomen and pelvis is a useful investigation for staging of the cancer rather than for initial diagnosis. A colonoscopy with biopsy is the gold-standard investigation for colorectal cancer, which is less likely in this case given the epigastric pain and melaena. Barium swallow is more useful for oesophageal motility disorders. Question 27 Answer: C. Mixed respiratory and metabolic acidosis Explanation: pH is low → acidosisPaCO₂ is high → respiratory acidosis HCO₃⁻ is low → metabolic acidosis This is not compensated as both systems are contributing to acidosis. Given his COPD and CKD, this is a mixed disorder. Question 28 Answer: D) Needle aspiration Explanation: This is a primary spontaneous pneumothorax in a young, otherwise healthy man. According to BTS guidelines, pneumothoraces >2 cm (or >50% collapse) in stable patients should be initially managed with needle aspiration. Chest drains are considered if aspiration fails. Needle decompression is for tension pneumothorax, which this is not (he is stable). High-flow oxygen is more useful post-procedure or for small asymptomatic pneumothoraces. Question 29 Correct answer: D. Roseola Infantum Explanation: Roseola is a common viral illness in infants and young children (6mo-3y), caused by HHV-6. Presents with 3-5 days of high fever. Maculopapular rash appear after the fever resolves. Measles usually begins with a prodrome of cough, coryza, and conjunctivitis, followed by a descending rash and Koplik spots. Scarlet fever presents with sore throat, fever, and a sandpaper-like rash. Rubella causes a milder rash with lymphadenopathy and typically affects older children. Erythema infectiosum causes a "slapped cheek" appearance, usually without high fever. Question 30 Answer: C. Venous ulcer Explanation: This is a classic venous ulcer, typically located over the medial malleolus, with irregular edges, shallow base, and associated signs of chronic venous insufficiency (e.g., haemosiderin deposition, lipodermatosclerosis). Arterial ulcers are usually painful, punched- out, and on pressure points like the toes. Question 31 Answer: B. Middle cerebral artery infarct Explanation: This patient has forehead sparing, which strongly suggests a UMN lesion. The middle cerebral artery supplies the motor cortex area responsible for the lower face (but the upper face gets bilateral input). Therefore, a stroke in this area causes contralateral lower face weakness with forehead sparing. LMN lesions (A, D and E) affect the entireipsilateral face, including the forehead (i.e. no forehead sparing). Brainstem glioma (C) is slow-growing and would likely cause other brainstem signs. Question 32 Answer: B Explanation: The patient is presenting with voiding symptoms such as hesitancy and terminal dribbling, which is suggestive of benign prostatic hyperplasia. DRE shows symmetrical enlargement and PSA is in the normal range, which rules out red flags and therefore 2-week wait referral (A) is not necessary. He has already tried lifestyle changes and these have failed, so it is appropriate to start pharmacological treatment with an alpha- blocker (e.g. tamsulosin). The GP should ideally review his symptoms in 4 to 6 weeks using the IPSS questionnaire. A TURP (C) and long-term catheterisation (E) are potential treatment options to be considered later if pharmacological management with an alpha blocker and 5- alpha reductase inhibitor also fail. Oxybutynin (D) would be appropriate as an additional drug if the patient had storage symptoms (e.g. urgency and frequency). Question 33 Answer: D. Dilated cardiomyopathy Explanation: This is a classic case of dilated cardiomyopathy, commonly caused by chronic alcohol abuse. Features suggestive of this include symptoms of global systolic dysfunction (S3, displaced apex beat) on examination and enlarged globular heart on heart, with a background history of alcahol use. Echocardiogram confirms the diagnosis. HOCM (A) usually has preserved systolic function and murmur, and might have a family history. Restrictive cardiomyopathy (B) usually has normal chamber size and presents with diastolic dysfunction. Constrictive pericarditis (C) would present with pericardial calcification and Kussmaul’s sign. PE (E) typically presents acutely with pleuritic chest pain, not chronically. Question 34 Answer: A Explanation: A thyroglossal cyst (A) can be identified on examination as a lump in the midline that moves up on tongue protrusion. B suggests a potentially malignant lesion. C suggests an inflamed reactive lymph node. D suggests a laryngocele, which becomes prominent with Valsava. E suggests a goitre that is characteristic of Graves disease, rather than a cyst. Question 35 Answer: B. Non-contrast CT Explanation: The patient has a sudden onset of focal neurology (facial weakness and sensory loss, as well as aphasia). This suggests stroke, which is likely to be a right middlecerebral artery stroke. Non-contrast CT head (B) is the first line investigation in order to exclude haemorrhage prior to considering thrombolysis. Thrombolysis (A) is wrong as imaging should be done before thrombolysis. Aspirin (C) is wrong as aspirin is given after haemorrhage is excluded. MRI (D) and CT angiogram (E) are wrong because these are not first line due to time and availability restrictions in the acute setting. However, they can be useful as subsequent steps if determining treatment steps (e.g. considering a thrombectomy). Question 36 Answer: C. Methotrexate Explanation: Methotrexate (C) is the first-line DMARD for rheumatoid arthritis due to its efficacy in reducing inflammation, halting disease progression, and preserving joint function. Anti-CCP positivity is highly specific for rheumatoid arthritis and indicates a more aggressive disease course. Hydroxychloroquine (A) and sulfasalazine (B) may be used in milder cases or as adjuncts. NSAIDs (D) and prednisolone (E) help manage symptoms but do not prevent joint damage long term. Question 37 Answer: C. Small cell lung cancer Explanation: This woman presents with proximal muscle weakness, dry mouth, weight loss, chronic cough, and a significant smoking history — all of which point towards a paraneoplastic syndrome secondary to small cell lung cancer (SCLC). The proximal muscle weakness and dry mouth are classic for Lambert-Eaton myasthenic syndrome (LEMS), a well-known paraneoplastic manifestation of SCLC. LEMS is caused by autoantibodies against presynaptic voltage-gated calcium channels, impairing acetylcholine release. SCLC is strongly associated with smoking and commonly presents with early systemic symptoms and paraneoplastic syndromes. Squamous cell lung cancer (A) and adenocarcinoma (E) are wrong as they are not typically associated with paraneoplastic syndromes. Tuberculosis (B) is wrong as there are no constitutional or TB-specific signs. Carcinoid tumour (D) is wrong because this usually causes serotonin-related symptoms. Question 38 Answer: C. NBM, NG tube, IV fluids, and close monitoring Explanation: This patient has signs of a small bowel obstruction likely due to post-operative adhesions (which is the most common cause in patients with surgical history). As there are no signs of ischaemia (normal lactate) or perforation (no free air), the initial management is conservative ("drip and suck"): keep NBM, insert a nasogastric tube for decompression, give IV fluids, and monitor electrolytes and urine output. Emergency laparotomy (A) and bowel resection with primary anastomosis (D) are reserved for signs of peritonism, sepsis, or failure of conservative treatment. Antibiotics/CT (B) are not indicated without sepsis. Sigmoidoscopy (E) is for large bowel decompression.Question 39 Answer: D. High-dose PPI and surveillance endoscopy every 2–3 years Explanation: The patient has non-dysplastic Barrett’s oesophagus >3 cm, which requires high-dose PPI therapy (e.g. lansoprazole) to suppress acid and prevent progression and surveillance endoscopy every 2–3 years (compared to 3–5 years for <3 cm Barrett’s). Endoscopic mucosal resection (A), endoscopic submucosal dissection (C) and radiofrequency ablation (E) are reserved for dysplasia or early carcinoma, for non-dysplastic cases. 6-month surveillance (B) is excessive for non-dysplastic Barrett’s. Question 40 Answer: C. Low factor VIII Explanation: This patient presents with prolonged superficial bleeding, normal platelets and low Factor VIII, which suggests von Willebran disease. VWF is needed to extend the half life of VIII, and so without it, levels are decreased. Low factor IX (A) is seen in haemophilia B. PT (B) is normal in vWD. Fibrinogen (D) and platelet count (E) are normal in vWD. Question 41 Answer: B. Serum anti-AChR antibody test Explanation: Myasthenia gravis is an autoimmune condition caused by antibodies targeting the acetylcholine receptor (AChR) at the neuromuscular junction. This leads to fatigable weakness, especially affecting the ocular and bulbar muscles, with ptosis and diplopia. The first-line diagnostic test is detection of serum anti-AChR antibodies. CT head (A) and MRI (C) are not helpful in confirming the diagnosis but may be used subsequently to exclude thymoma. Lumbar puncture (D) is useful in inflammatory or infectious CNS processes but not for myasthenia gravis. EEG (E) is indicated for seizure disorders. Question 42 Answer: C. Start duloxetine and optimise glycaemic control Explanation: This patient presents with classic diabetic peripheral neuropathy. The first-line management includes optimizing glycaemic control and using symptomatic treatments such as duloxetine or pregabalin for neuropathic pain. Nerve conduction studies (A) are not first- line unless the diagnosis is unclear. There is no indication for foot x-ray (B), neurology referral (D) or vitamin B12 injections (E) currently. Question 43 Answer: A. IdiopathicExplanation: The patient has left sided sudden sensorineural hearing loss (i.e. the cause will be in the inner ear). Whilst you worry about acoustic neuroma as this typically presents with unilateral tinnitus and hearing loss, the actual most likely cause is idiopathic (A). Acoustic neuromas (B) would present more gradually. Wax impaction (C) would cause a conductive hearing loss picture. The patient has no past medical history of note so is unlikely to be taking medications that could cause ototoxicity (D) such as antibiotics. Meniere’s (E) would present with vertigo and tinnitus in addition to hearing loss. Question 44 Answer: C Start a low dose inhaled corticosteroid Explanation: According to NICE guidelines, the next step after using a SABA (short-acting beta-agonist) in children is to initiate a regular low-dose inhaled corticosteroid (ICS) if symptoms persist. LTRA (B) and LABA (D) are considered after a trial of ICS if symptoms are still not controlled. Regular SABA (A) is not recommended. Oral corticosteroids (E) are reserved for acute exacerbations. Question 45 Answer: C. Intravenous 5% dextrose with gradual correction Explanation: This is hypernatraemic dehydration, likely from reduced intake. Urine osmolality is high which indicates ADH is working, and the kidneys are trying to retain water. Hypernatraemia must be corrected slowly with hypotonic fluids to avoid cerebral oedema, ideally no more than 8-10 mmol/L in 24 hours. 5% dextrose (C) is appropriate as a hypotonic solution to restore free water. Rapid correction (A) risks brain swelling. Oral water and high calorie diet (B) is not possible here as the patient is confused. Diuretics (D) are inappropriate here as they would cause further dehydration. Hypertonic saline (E) is used for hyponatraemia, not hypernatraemia. Question 46 Answer: C. Refer to clinical genetics Explanation: This patient presents with multiple clinical features highly suggestive of Neurofibromatosis Type 1 (NF1) — including more than 6 café-au-lait macules over 1 cm in diameter, axillary freckling, cutaneous neurofibromas, and mild scoliosis. These findings meet the NIH diagnostic criteria for NF1. Referral to clinical genetics is the most appropriate next step. This ensures diagnostic confirmation, genetic counselling, and access to multidisciplinary monitoring for associated complications, which can include learning difficulties, optic pathway gliomas, and skeletal abnormalities. Other options are inappropriate: Dermatology biopsy (B) is not necessary for diagnosis. MRI (A) may be indicated later based on symptoms, but not as an initial step. Corticosteroids (E) have no role in NF1 management. Reassurance or routine follow-up without formal diagnosis (D) risks missing complications that benefit from early intervention. Question 47Answer: D. Subdural haemorrhage Explanation: The patient is a classic presentation of a chronic subdural haemorrhage. The patient has risk factors such as a history of falls (i.e., prone to head injury) and atrial fibrillation (i.e., likely on anticoagulation given CHADSVASC score). Symptoms including confusion, personality changes and fluctuating levels of consciousness. An extradural haemorrhage would present differently, characteristically with a lucid interval on a history of head trauma followed by rapid deterioration. Frontotemporal dementia would not include fluctuating consciousness or acute confusion. A tumour, whilst possible, is less associated with the risk factors stated in the question stem making it a less likely diagnosis than a subdural haemorrhage. Ischaemic strokes have more acute onsets with focal neurological symptoms. Question 48 Correct answer: Prescribe phenoxymethylpenicillin Explanation: The presentation is consistent with bacterial tonsillitis. Patient score highly on Fever PAIN score of 5, hence suggest a bacterial cause and require immediate antibiotic. st Phenoxymethylpenicillin is the 1 line treatment for streptococcal tonsillitis. Amoxicillin is avoided initially as it may cause a rash in infectious mononucleosis (EBV). Azithromycin is considered in penicillin-allergic patient. Symptomatic treatment alone is appropriate for viral pharyngitis or low Centor score. Question 49 Answer: C Explanation: It is important to recognise that meningitis is a medical emergency and that the patient needs hospital treatment straight away. Therefore the most important thing the GP must do for this patient is call an ambulance. IM benzylpenicillin may be given if the patient is not penicillin allergic, however this is not a definitive treatment so option D is inappropriate. Question 50 Answer: B. Prednisolone 15 mg daily Explanation: Polymyalgia rheumatica (PMR) is a common inflammatory condition in older adults, presenting with proximal muscle stiffness and a raised ESR. It typically responds dramatically to low-dose corticosteroids, confirming the diagnosis. NSAIDs like naproxen are insufficient alone. Methotrexate is rarely used except in steroid-sparing strategies. Hydroxychloroquine is not effective for PMR. Physiotherapy may help long-term but is not a first-line treatment. Question 51Answer: C. Serum thyroid stimulating hormone Explanation: The patient’s symptoms are consistent with thyrotoxicosis, likely Graves' disease. The best initial investigation is measuring serum TSH, as it will be suppressed in primary hyperthyroidism. If necessary, free T4 levels can confirm the diagnosis. Question 52 Answer: D. JAK2 mutation testing Explanation: Essential thrombocythaemia (ET) is a JAK2-positive myeloproliferative neoplasm in ~50–60% of cases. A markedly raised platelet count in the absence of another cause. JAK2 mutation testing helps confirm the diagnosis. Question 53 Answer: B. IV artesunate Explanation: This is severe falciparum malaria, indicated by features such as jaundice and dark urine (suggestive of haemolysis or blackwater fever). First-line treatment is IV artesunate, which is highly effective and has been shown to be superior to IV quinine. Question 54 Answer: C – Constricted pupil Explanation: The pupil in acute angle closure glaucoma is typically dilated, not constricted. A mid-dilated pupil can block the irido-corneal angle, precipitating the rise in intraocular pressure. Vomiting, decreased vision, haloes, and hazy cornea are all common findings due to vagal stimulation and corneal oedema. Question 55 Answer: C. Squamous cell lung cancer Explanation: This patient’s symptoms (haemoptysis, weight loss, and a significant smoking history) are strongly suspicious for lung cancer. His haemoptysis and GI symptoms, in the context of weight loss and heavy smoking, point toward squamous cell carcinoma of the lung, which is a subtype of non-small cell lung cancer (NSCLC) that is strongly associated with smoking. Squamous cell lung cancer typically arises centrally, making it more likely tocause haemoptysis. Additionally, the electrolyte disturbances sometimes associated with squamous cell cancer (e.g. hypercalcaemia) can contribute to non-specific symptoms such as low mood and GI upset. While small cell lung cancer is also associated with smoking and paraneoplastic syndromes, it usually presents more aggressively and is more likely to cause endocrine symptoms like SIADH or Cushing’s syndrome. Pituitary adenoma would not explain the haemoptysis or smoking link, and carcinoid syndrome typically presents with flushing, diarrhoea, and wheeze, which are absent in this case. Question 56 Answer: C – Peripheral vasodilation and venous pooling Explanation: Vasovagal syncope is the most common type of fainting, particularly in adolescents and older adults. It occurs in response to triggers and results from a drop in heart rate and contractility, causing decreased cardiac output and peripheral vasodilation, which leads to venous pooling in the lower limbs. Question 57 Answer: D – Furosemide Explanation: This patient shows signs of hypovolemia and orthostatic hypotension: dry mucous membranes, reduced skin turgor, and a drop in BP with compensatory HR increase. These suggest over-diuresis, with furosemide — a loop diuretic — being the most likely cause. Question 58 Answer: A – Central retinal artery occlusion Explanation: The sudden, painless vision loss with a pale retina and a cherry-red spot is classic for CRAO. The afferent pupillary defect (RAPD) confirms optic nerve involvement. Other options (like CRVO or macular degeneration) present with different fundoscopic features or more gradual vision loss. Question 59 Answer: C. Widespread activation of coagulation cascade Explanation: This man is presenting with Disseminated Intravascular Coagulation, triggered by his sepsis. This results from increased activation of the coagulation (thrombi formed – elevated D-Dimer), then depletion of platelets and coagulation factors (bleeding, thrombocytopenia, low fibrinogen, prolonged PT and APTT). Question 60 Answer: D. Arrange urgent haemodialysis Explanation: This patient has refractory hyperkalaemia, severe metabolic acidosis, oliguria, and uraemic encephalopathy — all absolute indications for dialysis. She has already receivedfull medical treatment for hyperkalaemia. Continuing medical therapy alone is inappropriate. Diuretics are unlikely to work in true oliguria, and oral bicarbonate is too slow and ineffective in this acute setting. Question 61 Answer: A. Send blood for iron studies Explanation: Send blood for iron studies is the correct answer. The patient's blood test results indicate microcytic anaemia, which may indicate iron deficiency. While chronic kidney disease (CKD) can lead to anaemia due to diminished erythropoietin (EPO) production, it is imperative to assess the patient's iron status and initiate iron therapy prior to commencing treatment with erythropoiesis-stimulating agents (ESAs). Additionally, CKD may impair iron absorption because of increased hepcidin levels; hence, addressing any iron deficiency is essential as EPO will be less effective in its absence. Prescribe oral or IV EPO is inappropriate at this stage as EPO therapy should only be initiated after optimisation of iron status. EPO may be beneficial in enhancing physical function or quality of life for patients who are not suffering from iron deficiency. Prescribe IV iron supplementation is incorrect since diagnostic blood tests are necessary to confirm an iron deficiency before considering supplementation. Intravenous (IV) iron may be considered if target haemoglobin levels of 100-120 g/L are not reached following three months of oral supplementation. Send blood for erythropoietin level is also incorrect, as per NICE guidelines routine measurement of EPO levels is not recommended for the diagnosis or management of anaemia associated with CKD. Question 62 Answer: B. Oral antihistamines Explanation: This patient has acute urticaria, likely triggered by an allergen (shellfish). Oral antihistamines are the first-line treatment. Corticosteroids can be used in persistent or severe cases. Epinephrine is reserved for anaphylaxis, which this patient does not have. Question 63 Answer: A Explanation: b-HCG is a tumour marker for seminomas, we would expect it to be elevated in this patient. AFP is a tumour marker for non-seminomatous germ cell testicular cancers, however it is not raised in seminomatous cancers. Ca-125 is a tumour marker for ovarian cancer. Ca-19-9 is a tumour marker for pancreatic cancer. PSA is a tumour marker of prostate cancer. Question 64 Answer: D) 24-hour urinary catecholamines or plasma metanephrines Explanation: Any adrenal incidentaloma must be assessed for functionality and malignancyrisk. A key concern is ruling out a phaeochromocytoma, which can be asymptomatic but dangerous if missed. Therefore, plasma metanephrines or 24-hour urinary catecholamines are essential to screen for catecholamine secretion. In addition, a 1 mg overnight dexamethasone suppression test and aldosterone:renin ratio (in hypertensives) are also performed, but ruling out phaeochromocytoma is the top priority due to perioperative risks. Just monitoring the mass is inappropriate without proper initial endocrine workup. Question 65 Answer: A. Anterior packing and admit to hospital Explanation: The patient has epistaxis. First line would be the Hippocratic method which was already tried, cauterisation is the second line option so next doctors would consider anterior packing. Silver nitrate is what is used during cauterisation and posterior packing fourth line whilst surgery (arterial ligation) is only done after other options are exhausted. Question 66 Answer: D Explanation: Given the history of breastfeeding and the inflamed appearance of the lump, this is most likely a breast abscess. The treatment for a breast abscess is an urgent excision and drainage under the general surgeons. Flucoxacillin tends to be the antibiotic of choice for mastitis rather than co-amoxiclav. It is advisable to continue breastfeeding, however as a breast abscess needs to be drained, this isn’t the correct answer. Aspirin would be inappropriate as pain relief due the risk of Reye’s syndrome in infants. Question 67 Answer: C. CA 19-9 Explanation: According to Courvoisier’s Law, painless jaundice with a palpable gallbladder is more suggestive of a malignant pathology such as pancreatic cancer rather than a gallstone pathology. With the epigastric pain radiating to the back, unintentional weight loss and signs of obstructive jaundice, the most likely diagnosis in this case is pancreatic cancer. CA 19-9 is the most commonly associated tumour marker with pancreatic cancer. However, it can also be elevated in other pathologies such as chronic pancreatitis and cholestasis. AFP, CEA, CA 125, and chromogranin A are typically associated with hepatocellular carcinoma, colorectal cancer, ovarian cancer and neuroendocrine tumours respectively. Question 68 Answer: B – Osteoarthritis Explanation: This presentation is typical of osteoarthritis—mechanical joint pain, bony swelling, crepitus, and no systemic features. Rheumatoid and psoriatic arthritis are inflammatory and more symmetric with warmth or morning stiffness. Gout is usually monoarticular and acute.Question 69 Answer: C. Polycythaemia vera Explanation: Polycythaemia vera presents with hyperviscosity symptoms (headache, pruritus), raised Hb/Hct, and low EPO (distinguishing it from secondary causes). It is commonly associated with the JAK2 mutation. Dehydration could falsely raise Hb, but wouldn’t cause the symptoms or low EPO. Question 70 Answer: E. Thoracoscopy and biopsy Explanation: Mesothelioma is a malignant tumour of the pleura often related to asbestos exposure. While imaging (such as chest X-ray and CT scans) can suggest the diagnosis by showing pleural thickening, effusion, or masses, they are not definitive. Thoracoscopy (also known as video-assisted thoracoscopic surgery or VATS) allows direct visualisation of the pleura and enables targeted biopsy of affected tissue, making it the gold standard for confirming mesothelioma. Bronchoscopy may be useful in central lung lesions but often misses pleural pathology. CT chest and CT CAP are useful staging tools, and chest X-ray is too insensitive for definitive diagnosis. Question 71 Answer: C Silicosis Explanation: The presentation is suggestive of occupational lung disease – fine crackles is consistent with a restrictive lung pattern. His job as a construction worker expose him to inhaled dust/ fumes that predispose him to occupational lung disease. Bilateral small opacities on CXR is suggestive of silicosis, caused by inhaling silica dust, common in construction, mining workers. COPD is mainly caused by smoking and it is an obstructive lung disease. Asthma can be exacerbated by occupational exposure but will typically present with reversible airway obstruction, but not a restrictive pattern. TB can cause a chronic cough and weight loss, but the absence of night sweats, fever and haemoptysis makes it less likely. Known occupational exposure makes silicosis more likely than IPF, even though both presents similarly. Question 72 Answer: A Explanation: This patient is presenting to ED with an acute migrainous attack, as suggested by the aura and recurrent nature of her headaches. Sumatriptan is the most appropriate first line medication for an acute migraine. Morphine is not typically indicated as pain relief when managing migraines. The other three options are prophylactic treatments for migraines. Propranolol would be inappropriate for her as she has asthma and topiramate would be contraindicated as it is teratogenic and she is of childbearing age.Question 73 Answer: B Syndrome of inappropriate ADH secretion (SIADH) Explanation: This patient's findings, including low serum sodium and osmolality with inappropriately concentrated urine in a euvolaemic state, are consistent with SIADH. The left hilar mass suggests a possible small cell lung carcinoma, a known cause of SIADH. Question 74 Answer: B – Left circumflex artery Explanation: ST elevation in leads I, V5, and V6 indicates lateral wall myocardial infarction, which is most commonly due to occlusion of the left circumflex artery. Question 75 Answer: A – Coronary atherosclerosis Explanation: The slit-like tear in the anterior LV wall indicates free wall rupture, a known complication of transmural MI. This typically occurs 3–14 days post-MI and is often due to underlying coronary atherosclerosis. Question 76 Answer: B. Idiopathic pulmonary fibrosis Explanation: CXR shows ‘reticular’ (net-like) pattern of increased interstitial lung markings (opacities) in the peripheries of both lungs, worst at the lung bases (and volume loss). Alongside the history of dry cough, dyspnoea, clubbing and coal miner (occupational exposure), this is classic for idiopathic pulmonary fibrosis. Sarcoidosis (A) affects upper zones. COPD (C) shows hyperinflated lungs, flattened diaphragm. TB (D) causes cavitating lesions affecting upper zones. Bronchiectasis (E) may show ring shadows or tram-tracks but not diffuse fibrosis. http://www.svuhradiology.ie/case-study/pulmonary-fibrosis/ Question 77 Answer: C. Chest pain worse when lying down Explanation: Myocarditis often follows a viral illness and may present with flu-like symptoms, chest pain, and elevated cardiac enzymes due to myocardial inflammation. The chest pain can mimic pericarditis (pleuritic and worse when lying down). The other options (ankle oedema, ascites, Kussmaul’s sign, jaundice) are signs of chronic RHF or systemic disease, not typically acute myocarditis. Question 78 Answer: B. Systemic lupus erythematosus Explanation: SLE is a systemic autoimmune disease affecting multiple organs. Classicfeatures include mucocutaneous, musculoskeletal, renal, and hematologic involvement. Positive ANA is sensitive, and anti-dsDNA is specific, especially in lupus nephritis. RA affects joints symmetrically but lacks the skin and renal involvement. Sjögren’s typically causes sicca symptoms. Dermatomyositis presents with muscle weakness and distinctive rashes (e.g., heliotrope). Psoriatic arthritis is associated with psoriasis, nail changes, and asymmetric joint involvement. Question 79 Answer: B – Systemic lupus erythematosus Explanation: Joint pain, photosensitivity, fatigue, positive ANA and dsDNA, and renal involvement (proteinuria and red cell casts) strongly indicate SLE. RA lacks renal features; APS causes thromboses/miscarriages; dermatomyositis involves muscle weakness. Question 80 Answer: B. Offer cognitive behavioural therapy Explanation: This patient meets criteria for generalised anxiety disorder (GAD). CBT is first- line for mild to moderate cases, especially when patients prefer non-pharmacological approaches. SSRIs (C) are second-line or used if symptoms are more severe. Propranolol (A) is not first-line for GAD (more useful in performance anxiety), and benzodiazepines (D) are for short-term severe crises only. Question 81 Answer: A. Alteplase Explanation: The patient is said to have an ischaemic stroke. A CT scan has excluded a haemorrhage meaning that the patient can move on to treatment. As the individual has presented within 4.5 hours of symptom onset and has no contraindications, following nice guidelines, the patient would be given alteplase (thrombolysis). Aspirin would be given first line if it was >4.5 hours after symptom onset or would be given 24 hours following thrombolysis. An emergency thrombectomy would be performed if there was a suspected large vessel occlusion but also would not be performed as first line or before a patient has a CT angiogram. A statin is secondary prevention and not an immediate next step and clopidogrel is long-term prevention but is not given within the first 2 weeks following a stroke due to the risk of haemorrhagic transformation (i.e., a risk of the brain bleeding following a stroke). Question 82 Answer: C. MRI brain and spinal cord Explanation: Multiple sclerosis (MS) is diagnosed based on clinical presentation and evidence of demyelination disseminated in time and space. MRI is the gold standard investigation, allowing visualisation of white matter lesions in typical locations (periventricular, juxtacortical, infratentorial, and spinal cord). Visual evoked potentials maysupport the diagnosis but are not definitive. CT is less sensitive for white matter lesions. Lumbar puncture may show oligoclonal bands, useful as a supplementary test. Autoimmune blood screens help exclude other conditions but are not diagnostic for MS. Question 83 Answer: A. Grade 1 Explanation: Grade 1 pressure ulcers are characterized by non-blanching erythema on intact skin. Early recognition is essential to prevent progression. Grade 2 involves partial skin loss, Grade 3 involves full-thickness skin loss, and Grade 4 includes exposure of bone or muscle. Question 84 Answer: C Explanation: The components of a triple assessment include a clinical assessment, imaging of the breast (ultrasound or mammography) and biopsy (fine needle aspiration or core biopsy). The other tests mentioned may have a role in determining the spread of malignancy (sentinel LN excision, bone scan) or identifying risk factors (serum hormone concentrations, genetic testing). However, they are not part of the initial assessment for breast cancer. Question 85 Answer: E Explana on: The pa ent most likely has Paget’s disease, where excessive bone remodelling would be seen, leading to structurally abnormal, enlarged, and weakened bones. Bony enlargement of the pubic rami is a classic radiological finding in Paget’s disease. Neck of femur (NOF) fracture (A) and osteopenia (B) are more commonly associated with osteoporosis rather than Paget’s disease. Looser zones (C) are seen in osteomalacia, not Paget’s disease. Loss of joint space (D) is a sign of osteoarthri s, which does not explain this pa ent’s presenta on. Question 86 Correct answer: E Serum Lactate Explanation: In the initial management of suspected sepsis, serum lactate should be obtained first if this has not already been done, as this is a quick test and required for diagnosis and risk stratification. After this, blood culture blood cultures should be ideally obtained before administering antibiotics to help identify the causative organism and guide targeted antimicrobial therapy. Sepsis 6 include 1. Give oxygen 2. Take blood cultures 3. Give IV antibiotics 4. Give IV fluids 5. Measure lactate 6. Monitor urine outputQuestion 87 Answer: A. t(9;22) Explanation: The Philadelphia chromosome, t(9;22), is diagnostic for chronic myeloid leukaemia (CML). It results in the BCR-ABL1 fusion gene, a tyrosine kinase that drives the disease. Basophilia and a spectrum of immature and mature myeloid cells are characteristic of CML. Question 88 Answer: C. Contrast-enhanced CT chest Explanation: This man has Mackler’s triad (vomiting, chest pain, subcutaneous emphysema), pathognomonic for Boerhaave’s syndrome, a transmural oesophageal rupture. CT chest with oral water-soluble contrast (e.g., gastrografin) is the best initial investigation as it can detect the tear and mediastinal air. Barium is not preferred due to risk of mediastinitis if leakage occurs. Endoscopy is not recommended initially due to the risk of exacerbating the rupture. Question 89 Answer: D – Gonioscopy Explanation: Gonioscopy allows direct visualization of the anterior chamber angle and is diagnostic in acute angle closure glaucoma, which is suggested by the raised IOP, mid- dilated non-reactive pupil, and hypermetropia. Other tests (like fluorescein angiography or OCT) are not diagnostic for this condition. Question 90 Answer: B. IV acyclovir and IV ceftriaxone Explanation: The clinical picture and lymphocytic CSF with normal glucose are suggestive of viral encephalitis. HSV-1 is the most common cause. Empirical treatment includes acyclovir, but bacterial meningitis cannot be excluded, so ceftriaxone is also given. Question 91 Answer: C – Temporal artery biopsyExplanation: Clinical suspicion of giant cell arteritis warrants immediate steroid treatment, but biopsy confirms diagnosis. Imaging is not first-line. ESR and clinical signs guide initiation, biopsy confirms later. Question 92 Answer: B. Systemic inflammatory response syndrome (SIRS) and multiorgan dysfunction Explanation: This patient is developing complications from severe acute pancreatitis, particularly organ dysfunction (renal, respiratory, neurocognitive). The key underlying mechanism is SIRS, driven by enzyme leakage and inflammation, which causes widespread endothelial damage and capillary leak. Hypovolaemia plays a role but is not the primary driver here. Pseudocysts take longer (>1 week) to develop. Hypocalcaemia is common but not causing these systemic features. Question 93 Answer: B. Meniere’s disease Explanation: The patient has the typical triad for Meniere’s (vertigo, tinnitus and hearing loss). BPPV has no hearing loss or tinnitus. Acoustic neuromas are usually unilateral hearing loss that is progressive and could be associated with imbalance but vertigo is rare. Labyrinthitis is usually preceded by a viral infection and is not episodic. Vestibular neuritis would be a sudden onset of vertigo (without hearing loss) lasting days and it would not be episodic. Question 94 Answer: C. Vascular spasm and vasoconstriction Explanation: The first event following endothelial injury is vascular spasm, a reflex vasoconstriction that reduces blood flow and limits blood loss. This is mediated by the smooth muscle cells contracting and decreased production of NO and prostaglandins from the endothelial cells. Platelet adhesion, activation (thromboxane A2 and ADP), and aggregation follows. Activation of fibrinogen is not part of primary haemostasis. Question 95 Answer: B. IV hydrocortisone and IV fluids Explanation: This patient is experiencing an adrenal crisis—a medical emergency in which there is a critical deficiency of cortisol, often triggered by stress, infection, or missed steroid doses. Immediate management includes IV hydrocortisone (100 mg stat, then continued 6- hourly or via infusion) and aggressive IV fluid resuscitation with 0.9% saline to correct hypovolaemia and electrolyte disturbances. Oral steroids are inappropriate in an acute crisis. Fludrocortisone is used for maintenance in Addison’s but is not effective alone incrisis. Dexamethasone may be used if diagnosis is uncertain and cortisol testing is needed, but in known Addison’s, hydrocortisone is preferred. Antiemetics alone do not address the cortisol deficiency. Question 96 Answer: C Explanation: This man is presenting to ED with suspected testicular torsion based on his history of unresolving, severe pain, absent cremasteric reflex and Prehn’s sign on examination. This is a surgical emergency, therefore the most appropriate management is a surgical exploration as delay increases the likelihood of testicular ischaemia and necrosis. A scrotal USS can be used to confirm the diagnosis, but should not delay the patient going to theatre. A review by the day team is inappropriate due to the risk of testicular necrosis. This patient is also not fit enough for discharge. IV antibiotics would be appropriate if the patient was presenting with epididymitis. However, there is a negative history of sexually transmitted diseases and Prehn’s sign is negative (ie. testicular pain does not resolve on elevation of the scrotal sac), which makes torsion the most likely diagnosis. Question 97 Answer: A Decreased FVC, normal FEV1/ FVC ratio and decreased DLCO Explanation: The patient presents with symptoms and a high-resolution CT scan consistent with idiopathic pulmonary fibrosis (IPF), a restrictive disease. Restrictive lung disease has decreased FVC due to decreased lung expansion secondary to fibrosis. FEV1/FVC ratio may be normal or increased as both FVC and FEV1 are decreased proportionally but FEV1 tends to decrease less significantly than FVC. Diffusion capacity of lungs for carbon monoxide (DLCO) is decreased due to impaired gas exchange associated with loss of alveolar-capillary membrane surface area and fibrosis in IPF. Question 98 Answer: B. Bowel perforation Explanation: Abdominal x-ray reveals crescentic gas (free air) under the right diaphragm indicates pneumoperitoneum, most likely post-operative bowel perforation. Erect chest X- ray are often considered the best for detecting free subdiaphragmatic gas but an erect abdominal x-ray covering the lung bases will be just as sensitive. The AXR was likely ordered on suspicion of bowel obstruction. Paralytic ileus (A) and adhesions (C) cause distension but not free air. Ascites (D) is fluid, not gas. Volvulus (E) would show coffee-bean sign on AXR, not free air. https://radiopaedia.org/cases/bowel-perforation-subdiaphragmatic-free-gas Question 99 Answer: C. Hypoactive delirium Explanation: This patient shows an altered mental status, with normal observations and test results, as well as a triggering event (the fall). This alongside the withdrawn attitude shows hypoactive delirium. Dementia would be more gradual onset, there has been no indicatedhistory of depression, no hallucinations or delusions for a psychotic episode and a stroke would show neurological observations and symptoms. Question 100 Answer: E. Liver cysts Liver cysts develop in 80% of the patients diagnosed with ADPKD and is the most common extrarenal manifestation. In this scenario, the patient has persistent hypertension despite being on anti-hypertensive medications. This is accompanied by recurrent UTIs and an ultrasound scan showing multiple cysts on both kidneys. This patient is likely to be suffering from ADPKD. Hence, liver cysts will be the commonest extra-renal manifestation of ADPKD. Intracranial berry aneurysms only occurs in less than 16% of the patients. Mitral valve prolapse can be found in up to 25% of the patients which is less than liver cysts. Thyroid cysts are a very rare presentation of extra-renal manifestation in patients with ADPKD and will be less likely. Hydronephrosis is a common renal manifestation of ADPKD but is not an extra-renal manifestation. Question 101 Answer: A. Koebner phenomenon and Auspitz sign Explanation: The presentation is typical of psoriasis vulgaris. Koebner phenomenon refers to lesion development at sites of trauma; Auspitz sign is pinpoint bleeding when scale is removed. Question 102 Answer: E Staphylococcus aureus Explanation: The presentation is likely to be Hospital-acquired pneumonia. MRSA, Pseudomonas aeruginosa, and Enterobacteriaceae are more likely in HAP, but the most common HAP is MRSA. In contrast, Streptococcus pneumoniae and Mycoplasma pneumoniae are typical in community-acquired pneumonia. Mycoplasma is typically seen in atypical pneumonia. Question 103 Answer: C. CT scan of the chest, abdomen, and pelvis Explanation: This patient has rectal cancer confirmed by a colonoscopy with biopsy. Staging the tumour is an essential step to guide management and inform prognosis. A CT Chest, abdomen and pelvis is the most appropriate initial staging investigation to assess the extent of local invasion, lymph node involvement, and the presence of distant metastasis. A PET scan and endoscopic ultrasound can also be useful staging investigations but are not typically used first line. An abdominal X-ray will not allow for a clear visualisation of the tumour invasion.Question 104 Answer: B Explanation: The patient’s history is suggestive of idiopathic intracranial hypertension because of the postural nature of her headaches and risk factors, such as high BMI. Papilloedema is an examination sign that is associated with IIH. This is seen via fundoscopy as a blurring of the optic disc margins. A is a sign of central retinal artery occlusion. C describes drusen, which is associated with age-related macular degeneration. D and E are signs of hypertensive retinopathy. Question 105 Answer: C. Motor neuron disease (ALS) Explanation: ALS (amyotrophic lateral sclerosis) is the most common form of motor neuron disease and affects both upper and lower motor neurons. Symptoms include progressive limb weakness, fasciculations, hyperreflexia, and spasticity without sensory deficits. MS typically presents with sensory and motor deficits and is relapsing-remitting early on. Myasthenia gravis causes fatigable weakness without fasciculations or UMN signs. Guillain- Barré is an acute demyelinating condition causing ascending weakness with areflexia. Peripheral neuropathy usually presents with sensory symptoms first. Question 106 Answer: C. Granulomatosis with polyangiitis (Wegener’s) Explanation: GPA is a small-vessel vasculitis characterised by granulomatous inflammation involving the upper and lower respiratory tract and kidneys. c-ANCA (anti-PR3) is strongly associated. Microscopic polyangiitis is similar but lacks granulomas and ENT involvement. Goodpasture’s syndrome causes pulmonary-renal syndrome but is associated with anti- GBM antibodies. EGPA typically includes asthma and eosinophilia. SLE can cause nephritis and pulmonary issues but not ENT symptoms or c-ANCA positivity. Question 107 Answer: B. Zenker’s diverticulum Explanation: The patient presents with: Regurgitation of undigested food, halitosis, a lump- in-throat sensation. These are classic signs of Zenker’s diverticulum, a pharyngeal pouch just above the upper oesophageal sphincter. Achalasia causes dysphagia for solids and liquids. GORD presents with heartburn and acidic regurgitation—not undigested food. Gastritis causes pain, not regurgitation. Mallory-Weiss doesn’t cause regurgitation or halitosis. Question 108 Answer: E. IV lorazepam Explanation: The patient is in status epilepticus and has been seizing for over an hour. The patient is in A&E so IV access can be gained meaning IV lorazepam is the next best step as per guidelines. MRI and CT head would not be done as the patient is having an ongoingseizure and it is not the most important next step. IV phenytoin would be the next step if 2 doses of lorazepam 10 minutes apart did not stop the fit. Sodium valproate is not used for acute seizures but for seizure prevention. Question 109 Answer: A. t(9;22) Explanation: The clinical picture is indicative of chronic myeloid leukaemia (symptoms of abdominal fullness likely being due to splenomegaly and blood film with basophilia and leucocytosis). The translocation associated with CML is t(9;22) (q34; q11). For the other translocations t(8;14) is seen in Burkitt lymphoma, t(15;17) is seen in acute promyeloctic leukemia (APL), t(11;14) is seen in mantle cell lymphoma and t(14;18) is seen in follicular lymphoma. Question 110 Answer: C. Spirometry Explanation: Post-bronchodilator spirometry is indicated to confirm the diagnosis of COPD. The key diagnostic feature will be reduced FEV1/ FVC ratio which indicates an obstructive disease. CT is useful in evaluating lung damage and exclude other condition i.e. lung cancer but it is not 1 line investigation for COPD. CXR may help rule out other conditions i.e. pneumonia, but does not provide sufficient info to diagnose or assess the severity of COPD. ABG analysis is useful in assessing severity of respiratory failure in advanced COPD, but is not indicated in the initial diagnosis. Question 111 Answer: C. Osteoporosis Explanation: This patient has multiple low-impact fractures, which is highly suggestive of osteoporosis. The normal calcium, phosphate, ALP, and PTH levels support this diagnosis, as osteoporosis is typically a metabolic bone disorder with normal blood markers. Paget’s disease (A) would present with isolated high ALP and characteristic bony deformities. Multiple myeloma (B) causes hypercalcemia, renal impairment, and anaemia, which are absent here. Primary hyperparathyroidism (D) would show elevated calcium and PTH, which is not the case here. Bone mineral deficiency secondary to CKD (E) is unlikely, as this patient has normal phosphate, PTH, and vitamin D levels, which do not indicate renal bone disease. Question 112 Answer: A. Anastrozole Explanation: As this patient has an oestrogen-receptor positive breast cancer, she needs adjuvant endocrine therapy. Anastrozole and tamoxifen are both treatments given if the breast cancer is oestrogen-receptor positive. However, anastrozole is the more appropriate treatment in a woman who is post-menopausal, whereas tamoxifen is given to premenopausal women. Trastuzumab (Herceptin) is a biologic agent used in HER2-positive breast cancers. Radiotherapy is performed after a mastectomy if 4 or more lymph nodes are positive, the breast margins are positive/close or the tumour itself is larger than 5cm. However, this is not the case in this patient, so radiotherapy would not be necessary. Question 113 Answer: B. UltrasoundExplanation: The patient most likely has a thyroglossal cyst and to confirm this the doctors could perform an ultrasound. Immediate surgical referral would be premature. No symptoms suggest a bacterial cause and this antibiotics are not indicated. Further investigation would confirm the diagnosis this reassurance would be an inappropriate next step. Serum calcium would be indicated if there were parathyroid hormone imbalance related symptoms. Question 114 Answer: D – Psoriatic arthritis Explanation: Psoriasis with DIP involvement, nail pitting, dactylitis, and prolonged morning stiffness is characteristic of psoriatic arthritis. RA spares the DIPs; OA lacks systemic features; septic arthritis is acute and monoarticular. Question 115 Answer: B. Plasma metanephrines Explanation: This patient has features suggestive of multiple endocrine neoplasia type 2B (MEN 2B). One of the most significant risks in MEN 2B is phaeochromocytoma, and plasma metanephrines should be urgently tested before thyroid surgery to avoid a hypertensive crisis. Question 116 Answer: C – Emboli caused by atherosclerosis Explanation: The patient's focal neurological signs suggest a stroke. Her multiple cardiovascular risk factors point toward embolic stroke from atherosclerotic plaques as the underlying cause. Question 117 Answer: B – Renal disease Explanation: Renal disease is the most common secondary cause of hypertension due to its impact on the renin-angiotensin-aldosterone system. Conditions like polycystic kidney disease and diabetic nephropathy are common culprits. Question 118 Answer: B – Left retina or optic nerve Explanation: The swinging light test reveals a relative afferent pupillary defect (RAPD) in the left eye, indicating a lesion of the left retina or optic nerve, which impairs afferent input to the pupillary light reflex. Question 119 Answer: D. Empagliflozin Explanation: Empagliflozin is an SGLT2 inhibitor that promotes urinary glucose excretion, leading to weight loss and reduced cardiovascular risk, especially in patients with type 2 diabetes and established cardiovascular disease. It has proven benefits in reducing cardiovascular mortality. Question 120Answer: C. Percutaneous nephrostomy Explanation: This patient is presenting with an obstructed ureteric stone which has caused pyelonephritis, as suggested by his observations and enlarged (likely oedematous) right kidney on the CT KUB. The priority here is to allow the right kidney to drain urine, in order to resolve the infection and reduce the extent of kidney injury. A percutaneous nephrostomy is the insertion of a catheter into the kidney through the skin, which allows urine to bypass the stone and drain into a collection bag. The ureteric stone will later be removed by percutaneous nephrolithotomy (PNCL). ESWL and ureteroscopy are both procedures suitable for stones under 2cm. While options B and E would play a role in this patient’s management, they would not reduce his immediate risk of kidney injury. Question 121 Answer: C. Recombinant factor VIII Explanation: Haemophilia A is due to factor VIII deficiency. In cases of significant bleeding like hemarthrosis (bleeding into joints), the definitive treatment is replacement of factor VIII. Desmopressin (DDAVP) can be used in mild cases, as it releases stored vWF and factor VIII, but this is not enough to treat a hemarthrosis. FFP contains all the clotting factors, so would be effective, but is a waste of resources when there is only a fault with FVIII. Platelet transfusion and IVIG would not be indicated. Question 122 Answer: C. Echocardiography Explanation: The key signs (RHF, Kussmaul’s sign, hepatomegaly) suggest constrictive pericarditis or restrictive cardiomyopathy. Echo is the best initial investigation to assess pericardial thickness, ventricular filling, and to distinguish between the two. Biopsy is rarely used initially and is invasive. BNP can support HF diagnosis but lacks specificity. Abdominal ultrasound is not useful for identifying the cardiac cause, and MRI is 2 line after echo. Question 123 Answer: B. Sjögren’s syndrome Explanation: Sjögren’s is an autoimmune exocrinopathy presenting with sicca symptoms (dry eyes and mouth), positive Schirmer’s test, and anti-Ro/La antibodies. RA may coexist but primarily affects joints. Systemic sclerosis involves skin thickening and Raynaud’s. Polymyositis causes muscle weakness. Sarcoidosis is a granulomatous disease often affecting the lungs, not classically associated with sicca symptoms. Question 124 Answer: B. Immediate thrombolysis Explanation: This is a massive (high-risk) pulmonary embolism, presenting with haemodynamic instability and signs of right heart strain. In such cases, thrombolysis is indicated immediately, even before definitive imaging. Heparin alone is insufficient in haemodynamic collapse. CTPA is not required before treatment when PE is clinically certain and life-threatening. Question 125Answer: D. Oral prednisolone 40–60 mg Explanation: Giant cell arteritis (GCA) is a large vessel vasculitis that can lead to irreversible vision loss if untreated. First-line treatment is high-dose corticosteroids initiated immediately. Oral prednisolone 40–60 mg daily is recommended unless there is visual loss, which may require IV methylprednisolone. NSAIDs and paracetamol are not effective treatments. Low-dose aspirin may be added but is not first-line. Methotrexate may be considered in refractory or relapsing cases as a steroid-sparing agent. Question 126 Answer: C. Pre-renal AKI due to ACE inhibitor + diuretic Explanation: This patient likely has pre-renal AKI due to a combination of volume depletion (from furosemide) and efferent arteriole vasodilation (from ramipril), which reduces glomerular filtration pressure. His low JVP, hypotension, and bland urinalysis support this. There is no evidence of intrinsic renal disease (e.g., no active urinary sediment), and ultrasound excludes obstruction. ATN usually follows prolonged hypoperfusion and has a different urinalysis pattern. AIN would typically have eosinophils and possibly rash/fever. Question 127 Answer: A. Hepatocellular carcinoma Explanation: This patient has presented with right upper quadrant abdominal pain, jaundice and weight loss. Coupled with the risk factors of hepatitis C and cirrhosis, the most likely diagnosis in this case is hepatocellular carcinoma. This is further supported by the elevated tumour marker AFP. The location of the right upper quadrant pain makes gastric and pancreatic cancer less likely which typically present with epigastric pain. Gallbladder and cholangiocarcinoma are important differentials, however the elevated AFP levels, makes hepatocellular carcinoma more likely. Question 128 Answer: D – Gout Explanation: The classic finding of negatively birefringent needle-shaped crystals on joint aspiration confirms gout. Pseudogout has positively birefringent rhomboid crystals. Septic arthritis shows organisms, not crystals. Question 129 Answer: C. IV 0.9% saline and reassess potassium Explanation: This patient presents with diabetic ketoacidosis (DKA), which requires immediate fluid resuscitation to correct dehydration. Insulin therapy should only be initiated once potassium levels are sufficient (>3.5 mmol/L), as insulin can drive potassium into cells, potentially worsening hypokalemia. Question 130 Answer: D. Admit into hospital with IV antibiotics Explanation: The presentation suggests orbital cellulitis, a medical emergency. The presence of proptosis, ophthalmoplegia, fever, and recent sinus infection supports the diagnosis. This requires urgent IV antibiotics and inpatient management to prevent spread to the brain or optic nerve.Question 131 Answer: C. Prescribe 20 mg atorvastatin Explanation: Given the patient’s age and persistent dyslipidaemia despite lifestyle changes, atorvastatin 20 mg is the appropriate moderate-intensity statin for primary prevention per NICE guidelines. Question 132 Answer: E. 7.7 mmol/L Explanation: Total cholesterol ≥7.5 mmol/L strongly suggests familial hypercholesterolaemia, particularly in patients showing physical signs like corneal arcus and xanthomata. Question 133 Answer: A. Ramipril Explanation: Ramipril, an ACE inhibitor, can cause angioedema — a rare but serious adverse effect caused by bradykinin accumulation. This swelling can occur even months after starting therapy. Question 134 Answer: A. 1st degree heart block Explanation: 1st degree heart block is characterized by a PR interval > 0.2 seconds with no dropped beats (no missed QRS complexes). This matches the ECG findings given. Question 135 Answer: C. Streptococcus pneumoniae Explanation: While HIV-positive patients are at increased risk of opportunistic infections like Pneumocystis jirovecii, this typically occurs in advanced immunosuppression (CD4 <200) and presents with diffuse bilateral interstitial infiltrates, not focal findings. This patient has focal signs on examination (dullness to percussion in one lung zone), suggesting lobar consolidation, which is most characteristic of typical bacterial pneumonia. In this context, Streptococcus pneumoniae is the most common causative organism of community-acquired pneumonia, including in HIV-positive individuals. Thus, it remains the most likely pathogen in this case. Question 136 Correct Answer: D. Intravenous naloxone Explanation: The patient presents with classic signs of opioid overdose: respiratory depression, pinpoint pupils (miosis), and decreased consciousness, especially with IV drug use (track marks). - Naloxone, an opioid antagonist, rapidly reverses opioid effects, especially respiratory depression. - IV route provides faster onset than IM or oral. - Oral naltrexone is for long-term relapse prevention, not acute overdose. - Thiamine is for Wernicke’s encephalopathy. - CPR or intubation are used if naloxone fails or in extreme cases, but naloxone is the first step. Question 137 Answer: B. Initiate infliximab Explanation: In this steroid-refractory Crohn’s patient with active fistulising perianal disease, infliximab is indicated because anti-TNF therapy rapidly induces remission and promotes fistula closure, whereas azathioprine or methotrexate act slowly and are more appropriate for maintenance rather than acute control. Seton placement alone addresses only drainage and does not treat underlying inflammation, so it must be combined with medical therapy rather than used in isolation. Question 138 Correct Answer: B. Chlordiazepoxide Explanation: The patient is showing early signs of alcohol withdrawal (tremors, anxiety, sweating). Chlordiazepoxide, a long-acting benzodiazepine, is the first-line treatment due to smoother and safer withdrawal control. Diazepam is also a benzodiazepine but less commonly used in this context. Carbamazepine and phenytoin are antiepileptics, not first-line for alcohol withdrawal. Supportive care alone is inadequate due to risks of seizures or delirium tremens. Question 139 Answer: B. Gluten-free diet Explanation: The definitive management of biopsy-proven celiac disease is strict gluten avoidance, which leads to mucosal healing and resolution of malabsorption; a low-FODMAP diet targets functional IBS symptoms rather than immune-mediated villous atrophy, pancreatic enzyme replacement treats exocrine pancreatic insufficiency, not villous damage, and while lactose restriction may help secondary intolerance, it does not address the root cause. Question 140 Answer: C. Amoxicillin 7 days Explanation: In pregnant women with a UTI: First line: Nitrofurantoin for 7 days (however is contraindicated near term); Second line: Amoxicillin or Cefalexin for 7 days. Nitrofurantoin is contraindicated in the last trimester in pregnancy due to the risk of neonatal haemolytic anaemia. Avoid Trimethoprim altogether as it is teratogenic.Question 141 Answer: D Explanation: This patient has hypercalcemia in the setting of small cell lung cancer, which is likely PTHrP-mediated. Thiazide diuretics (hydrochlorothiazide) exacerbate hypercalcemia by increasing calcium reabsorption in the kidneys. Therefore, stopping the thiazide diuretic and promoting hydration to enhance calcium excretion is the best approach. Oral alendronate (A) is used for osteoporosis, not acute hypercalcemia. Encouraging oral fluids alone (B) is helpful but insufficient without stopping the exacerbating medication. IV rehydration (C) is reserved for severe hypercalcemia with symptoms. Denosumab (E) would be used in refractory cases but is not first-line. Question 142 Correct Answer: B. Administer IM adrenaline Explanation: The patient is experiencing anaphylaxis, characterized by urticarial rash, wheeze, and hypotension following exposure to a known allergen (peanuts). IM adrenaline (into the lateral thigh) is the first-line treatment for anaphylaxis. IV adrenaline is used in advanced care settings with close monitoring. According to Resuscitation Council UK guidelines: • Call for help • Remove the trigger • Lie the patient flat • Then administer IM adrenaline High-flow oxygen and antihistamines are supportive but not first-line treatments. Question 143 Answer: C. Conservative observation with hydration and NSAIDs Explanation: In presumed viral mesenteric adenitis, characterised by RLQ pain, low-grade fever, benign imaging of a normal appendix with enlarged lymph nodes, supportive care alone is indicated; surgery is unwarranted without appendiceal pathology, antibiotics are unnecessary in a likely viral process, and invasive biopsy is excessive in a self-limited condition. Question 144 Answer: C. Mesenteric angiography via CT Explanation: The classic finding of pain out of proportion to examination, along with AF and raised lactate, is highly suggestive of acute mesenteric ischaemia, typically due to embolism to the superior mesenteric artery. The gold standard for diagnosis is CT angiography, whichis non-invasive and rapidly identifies vascular occlusion and bowel viability. Ultrasound is less sensitive, especially for bowel. Erect CXR and AXR are not helpful early. Laparoscopy may be therapeutic but is not first-line for diagnosis. Question 145 Answer: B. Trial of a low-FODMAP diet and fibre supplementation Explanation: Her presentation meets IBS criteria in the absence of alarm features or laboratory abnormalities; initial management emphasises dietary modification, specifically reducing fermentable carbohydrates, and soluble fibre to regulate bowel habits, whereas colonoscopy, CT imaging, or empiric antibiotics are reserved for patients with red flags, refractory symptoms, or predominant bloating not responsive to first-line measures. Question 146 Correct Answer: B. Vitamin B3 deficiency Explanation: The clinical picture is consistent with pellagra, which is due to niacin (vitamin B3) deficiency, commonly seen in individuals with chronic alcoholism and poor nutrition. Pellagra is characterized by the triad of: • Dermatitis (especially photosensitive) • Diarrhoea • Dementia Other B-vitamin deficiencies have different presentations: • B1 (thiamine): Wernicke’s encephalopathy • B6: Peripheral neuropathy • B9 and B12: Macrocytic anaemia, with neurological features but not pellagra symptoms Question 147 Answer: D. JAK2 Explanation: Around 95% of all patients with polycythaemia vera have the JAK2 mutation. BCR ABL mutation is associated with CML. ANK1 is associated with hereditary spherocytosis. TET2 is associated with polycythaemia vera, but it is less specific and sensitive than JAK2. None of these would present with anaemia, not the characteristic itching after hot baths. Question 148 Answer: B. Incarceration leading to strangulation Explanation: The location of the lump inferolateral to the pubic tubercle indicates a femoral hernia, which is more common in elderly females and has a high risk of strangulation due to the narrow femoral canal. Strangulation leads to ischaemia and necrosis of trapped bowel,making emergency surgery essential. Obstruction can occur but strangulation is more urgent and dangerous. The other options are anatomically or pathophysiologically unrelated. Question 149 Correct Answer: D. Remove the sting and apply cold compress Explanation: This is a localised wasp sting without systemic or allergic symptoms. Management involves conservative treatment: • Remove the sting if visible (though wasps often don’t leave one) • Apply a cold compress to reduce swelling and discomfort • Adrenaline is unnecessary unless anaphylaxis is present. • Antibiotics are not indicated unless infection occurs. • Tetanus immunoglobulin is only needed for dirty wounds or unknown vaccination history. • Tourniquets are contraindicated. Question 150 Answer: C. Urgent incision and drainage Explanation: A tender, fluctuant perianal swelling accompanied by systemic signs is diagnostic of a perianal abscess, which requires prompt incision and drainage for source control and pain relief; conservative measures like sitz baths or fibre do not resolve pus collections, antibiotics without drainage fail to address the nidus of infection, and imaging delays treatment when the clinical diagnosis is clear.